CCRI Exam IV

Lakukan tugas rumah & ujian kamu dengan baik sekarang menggunakan Quizwiz!

What is the most common complication of a Balloon Tamponade Sengstaken-Blakemore?

Aspiration pneumonia (PNA)

A nurse should monitor for these symptoms in a patient suspected of acute Pancreatitis?

Paralytic ileus, bowel sounds, rebound, guarding of abdomen.

What is a normal Serum Amylase level?

30-220 units/L

A nurse cares for a client who is prescribed 4 mg of calcium gluconate to infuse over 5 hours. The pharmacy provides 2 premixed infusion bags with 2 mg of calcium gluconate in 100 mL of D5W. At what rate should the nurse administer this medication? (Record your answer using a whole number.) ____ mL/hr

40 mL/hr

An emergency department nurse cares for a client who is severely dehydrated and is prescribed 3 L of intravenous fluid over 6 hours. At what rate (mL/hr) should the nurse set the intravenous pump to infuse the fluids? (Record your answer using a whole number.) ____ mL/hr 500 mL/hr Because IV pumps deliver in units of milliliters per hour, the pump would have to be set at 500 mL/hr to deliver 3 L (3000 mL) over 6 hours. 6x = 3000 x = 500

500 mL/hr

How does a patient get Acute Pancreatitis?

#1: Biliary Tract Disease (gallstones) #2: ETOH abuse (edema and swelling of pancreatic duct) #3: Trauma, viral infections. Hypertriglyceridemia

What are the clinical symptoms of Acute Pancreatitis?

*Abdominal Pain (Mid-epigastric, LUQ, severe, steady) Vomiting/Nausea Fever Jaundice Hypotension Cullen's sign Turner's sign Dyspnea Elevated Serum Amylase/Lipase

What is a normal serum lipase level?

0-160 units/L

What temperature heat the Dialysate?

100 degrees F or 37.8 C

A nurse cares for a client with acute pancreatitis who is prescribed gentamicin (Garamycin) 3 mg/kg/day in 3 divided doses. The client weighs 264 lb. How many milligrams should the nurse administer for each dose? (Record your answer using a whole number.) ____ mg/dose ANS: 120 mg/dose 264 lb ÷ (2.2 lb/kg) = 120 kg. 3 mg/kg/day ´ 120 kg = 360 mg/day. 360 mg/day ÷ 3 divided doses = 120 mg/dose.

120 mg

What is a normal Calcium (Ca2+) level?

9.0-10.5 mg/dL

A client diagnosed with schizophrenia receives fluphenazine decanoate (Prolixin Decanoate) from a home health nurse. The client refuses medication at one regularly scheduled home visit. Which nursing intervention is ethically appropriate? A. Allow the client to decline the medication and document the decision. B. Tell the client that if the medication is refused, hospitalization will occur. C. Arrange with a relative to add the medication to the client's morning orange juice. D. Call for help to hold the client down while the injection is administered. Rationale: It is ethically appropriate for the nurse to allow the client to decline the medication and provide accurate documentation. The client's right to refuse treatment should be upheld, unless the refusal puts the client or others in harm's way.

A

A client is in the emergency department after a motor vehicle crash. In assessing the client, which clinical sign alerts the nurse to the presence of possible liver trauma? a.Abdominal pain referred to the right shoulder b.Left upper quadrant abdominal pain and swelling c. Abdominal pain referred to the spine and legs d. Abdominal pain with accompanying rebound tenderness One of the key features of liver trauma is abdominal pain that is increased on deep breathing and is referred to the right shoulder. The liver is on the right, not the left, side of the body. Liver injury does not produce pain that radiates to the spine and down the legs. Rebound tenderness can indicate peritonitis.

A

A male client comes into the emergency department with a serum creatinine of 2.2 mg/dL and a blood urea nitrogen (BUN) of 24 mL/dL. What question should the nurse ask first when taking this client's history? a. "Have you been taking any aspirin, ibuprofen, or naproxen recently?" b. "Do you have anyone in your family with renal failure?" c. "Have you had a diet that is low in protein recently?" d. "Has a relative had a kidney transplant lately?" There are some medications that are nephrotoxic, such as the nonsteroidal anti-inflammatory drugs ibuprofen, aspirin, and naproxen. This would be a good question to initially ask the client since both the serum creatinine and BUN are elevated, indicating some renal problems. A family history of renal failure and kidney transplantation would not be part of the questioning and could cause anxiety in the client. A diet high in protein could be a factor in an increased BUN.

A

A nurse assesses a client who is recovering from a Whipple procedure. Which assessment finding alerts the nurse to urgently contact the health care provider? a. Drainage from a fistula b. Absent bowel sounds c. Pain at the incision site d. Nasogastric (NG) tube drainage Complications of a Whipple procedure include secretions that drain from a fistula and peritonitis. Absent bowel sounds, pain at the incision site, and NG tube drainage are normal postoperative findings.

A

A nurse cares for a client with an increased blood urea nitrogen (BUN)/creatinine ratio. Which action should the nurse take first? a. Assess the client's dietary habits. b. Inquire about the use of nonsteroidal anti-inflammatory drugs (NSAIDs). c. Hold the client's metformin (Glucophage). d. Contact the health care provider immediately. An elevated BUN/creatinine ratio is often indicative of dehydration, urinary obstruction, catabolism, or a high-protein diet. The nurse should inquire about the client's dietary habits. Kidney damage related to NSAID use most likely would manifest with elevations in both BUN and creatinine, but no change in the ratio. The nurse should obtain more assessment data before holding any medications or contacting the provider.

A

The nurse monitors for which clinical manifestation in a client with a decreased fecal urobilinogen concentration? a.Clay-colored stools b.Petechiae c.Asterixis d.Melena When fecal urobilinogen levels are decreased as a result of biliary cirrhosis, the stools become lighter or clay-colored.

A

A nurse contacts the health care provider after reviewing a client's laboratory results and noting a blood urea nitrogen (BUN) of 35 mg/dL and a creatinine of 1.0 mg/dL. For which action should the nurse recommend a prescription? a. Intravenous fluids b. Hemodialysis c. Fluid restriction d. Urine culture and sensitivity Normal BUN is 10 to 20 mg/dL. Normal creatinine is 0.6 to 1.2 mg/dL (males) or 0.5 to 1.1 mg/dL (females). Creatinine is more specific for kidney function than BUN, because BUN can be affected by several factors (dehydration, high-protein diet, and catabolism). This client's creatinine is normal, which suggests a non-renal cause for the elevated BUN. A common cause of increased BUN is dehydration, so the nurse should anticipate giving the client more fluids, not placing the client on fluid restrictions. Hemodialysis is not an appropriate treatment for dehydration. The lab results do not indicate an infection; therefore, a urine culture and sensitivity is not appropriate.

A

A nurse evaluates a client with acute glomerulonephritis (GN). Which manifestation should the nurse recognize as a positive response to the prescribed treatment? a. The client has lost 11 pounds in the past 10 days. b. The client's urine specific gravity is 1.048. c. No blood is observed in the client's urine. d. The client's blood pressure is 152/88 mm Hg. Fluid retention is a major feature of acute GN. This weight loss represents fluid loss, indicating that the glomeruli are performing the function of filtration. A urine specific gravity of 1.048 is high. Blood is not usually seen in GN, so this finding would be expected. A blood pressure of 152/88 mm Hg is too high; this may indicate kidney damage or fluid overload.

A

A nurse reviews the allergy list of a client who is scheduled for an intravenous urography. Which client allergy should alert the nurse to urgently contact the health care provider? a. Seafood b. Penicillin c. Bee stings d. Red food dye Clients with seafood allergies often have severe allergic reactions to the standard dyes used during intravenous urography. The other allergies have no impact on the client's safety during an intravenous urography.

A

A physician orders methylphenidate (Ritalin) for a child diagnosed with ADHD. Which information about this medication should the nurse provide to the parents? A. If one dose of Ritalin is missed, double the next dose. B. Administer Ritalin to the child after breakfast. C. Administer Ritalin to the child just prior to bedtime. D. A side effect of Ritalin is decreased ability to learn. Rationale: The nurse should instruct the parents to administer Ritalin to the child after breakfast. Ritalin is a central nervous system stimulant and can cause decreased appetite. Central nervous system stimulants can also temporarily interrupt growth and development.

A

The nurse reviews laboratory results for a client with cirrhosis and finds the following: hematocrit, 72%; blood urea nitrogen (BUN), 42 mg/dL; and sodium, 166 mEq/L. Which action by the nurse is most appropriate? a.Check the client's blood pressure and pulse. b.Increase the client's oral fluid intake. c.Call the health care provider. d.Document the results in the chart. These values are all elevated, which can occur in hypovolemia. The nurse should assess the client for signs of hypovolemia, including tachycardia and hypotension. The nurse should consult with the provider about the client's fluid status before increasing oral fluids but after obtaining vital signs. Documentation should occur after all assessments have been completed and must include actions taken.

A

Which intervention will the nurse include in the plan of care for a client with severe liver disease? A. Encourage the client to eat a low-protein, high-carbohydrate diet. B. Administer Kayexalate enemas. C. Instruct the client to eat a high-protein, low-carbohydrate diet. D. Teach the client to participate in frequent, vigorous physical activities.

A

What is a Pancreatic Abscess?

A cavity within the pancreas, contains fluid, results from extensive necrosis in the pancreas, may perforate, leads to sepsis.

What is Venous thromboembolism (VTE)?

A term that refers to both deep vein thrombosis and pulmonary embolism; obstruction by a thrombus.

A nurse plans care for a client who has acute pancreatitis and is prescribed nothing by mouth (NPO). With which health care team members should the nurse collaborate to provide appropriate nutrition to this client? (Select all that apply.) a. Registered dietitian b. Nursing assistant c. Clinical pharmacist d. Certified herbalist e. Health care provider Clients who are prescribed NPO while experiencing an acute pancreatitis episode may need enteral or parenteral nutrition. The nurse should collaborate with the registered dietitian, clinical pharmacist, and health care provider to plan and implement the more appropriate nutritional interventions. The nursing assistant and certified herbalist would not assist with this clinical decision.

A, C, E

A client asks the nurse, "What are the advantages of peritoneal dialysis over hemodialysis?" Which response by the nurse is accurate? (Select all that apply.) a."It will give you greater freedom in your scheduling." b."You have less chance of getting an infection." c."You need to do it only three times a week." d."You do not need a machine to do it." e."You will have fewer dietary restrictions." Although peritoneal dialysis is slower than hemodialysis, it does not require a specially trained registered nurse and can be done at home, allowing for greater flexibility in scheduling. Peritoneal dialysis is ambulatory, and a machine is not needed. Nursing implications for hemodialysis include vascular access care and diet restrictions, whereas peritoneal dialysis allows for a more flexible diet (abdominal catheter care is still necessary).

A, D, E

A client had an open Whipple procedure yesterday for pancreatic cancer. Which nursing interventions are appropriate for this client? (Select all that apply.) A. Monitor and document the client's nasogastric tube drainage. B. Place the client in a side-lying position to promote wound drainage. C. Assess the abdomen for signs of peritonitis. D. Monitor the client's hemoglobin and hematocrit. E. Check the client's blood glucose frequently.

A,C,D,E

What is Acute Pancreatitis?

An acute inflammation of the pancreas, extent of inflammation ranges from mild to severe.

What diagnostic tests can aid in finding Pancreatic Cancer?

Abdominal ultrasound and contrast-enhanced CT confirming a tumor and can differentiate the tumor from a cyst.

What can an elevated Serum Lipase level mean?

Acute pancreatitis

What is a Splenomegaly?

An enlarged spleen results from the backup of blood into the spleen. The enlarged spleen destroys platelets, causing thrombocytopenia (low serum platelet count) and increased risk for bleeding.

How is liver transplant rejection treated?

Aggressively treated with immunosuppressive drugs. As with all rejection treatments, the patient is at a greater risk for infection. The only alternative for treatment is emergency retransplantation.

What is Hepatic Encephalopathy?

Also, called portal-systemic encephalopathy [PSE]) is a complex cognitive syndrome that results from liver failure and cirrhosis. Ammonia enters systemic circulation without liver detoxification. Ammonia crosses blood/brain barrier causes neurologic toxic manifestations. Patients report sleep disturbance, mood disturbance, mental status changes, and speech problems early as this complication begins.

What is Hepatitis C?

An enveloped, single-stranded RNA virus. Transmission is blood to blood. The rate of sexual transmission is very low in a single-couple relationship but increases with multiple sex partners. Unlike with hepatitis B, most people infected with hepatitis C do not clear the virus and a chronic infection develops, usually does its damage over decades by causing a chronic inflammation in the liver that eventually causes the liver cells to scar. This scarring may progress to cirrhosis

DKA S&S

Anorexia n/v (nausea and vomiting) polyuria polydipsia severe dehydration hypovolemic shock classic signs Kussmaul respirations, acetone breath (sweet fruity odor) ketonuria, dry mucous membranes weakness fatigue, H/A (headache) hypotension

What make a variceal bleed worse?

Any activity that increases abdominal pressure may increase the likelihood of a variceal bleed, including heavy lifting or vigorous physical exercise. In addition, chest trauma or dry, hard food in the esophagus can cause bleeding.

The two most common complications of a liver transplant rejection are?

Are acute graft rejection and infection.

What are Esophageal varices?

As a result of portal hypertension, the blood backs up from the liver and enters the esophageal and gastric veins. Fragile, thin-walled esophageal veins become distended and tortuous from increased pressure. The potential for varices to bleed depends on their size; size is determined by direct endoscopic observation. Varices occur most often in the distal esophagus but can be present also in the stomach and rectum.

What medications are approved for ADHD?

Atomoxatine (Strattera) Bupropion (Wellbutrin) Ritalin (methylphenidate) Adderall (amphetamine)

A child has been recently diagnosed with mild IDD. What information about this diagnosis should the nurse include when teaching the child's mother? A. Children with mild IDD need constant supervision. B. Children with mild IDD develop academic skills up to a sixth-grade level. C. Children with mild IDD appear different from their peers. D. Children with mild IDD have significant sensory-motor impairment. Rationale: The nurse should inform the child's mother that children with mild IDD develop academic skills up to a sixth-grade level. Individuals with mild IDD are capable of independent living, capable of developing social skills, and have normal psychomotor skills.

B

A client has cirrhosis. Which nursing intervention would be most effective in controlling ascites? a.Monitoring intake and output b.Providing a low-sodium diet c.Increasing oral fluid intake d.Weighing the client daily A low-sodium diet is one means of controlling abdominal fluid collection. Monitoring intake and output does not control fluid accumulation, nor does weighing the client. These interventions merely assess or monitor the situation. Increasing fluid intake would not be helpful.

B

A client is admitted with cirrhosis and hepatopulmonary syndrome. Which clinical manifestation does the nurse monitor for progression or resolution of this problem? a. Right upper quadrant pain b. Crackles on auscultation c. Skin and scleral jaundice d. Nausea and vomiting An increase in intra-abdominal pressure from ascites can lead to hepatopulmonary syndrome. This is manifested by dyspnea and crackles on auscultation. The other symptoms are consistent with liver disease but are not specific to respiratory involvement.

B

A client is recovering from a kidney transplant. The client's urine output was 1500 mL over the last 12-hour period since transplantation. What is the priority assessment by the nurse? a. Checking skin turgor b. Taking blood pressure c. Assessing lung sounds d. Weighing the client By taking blood pressure, the nurse is assessing for hypotension that could compromise perfusion to the new kidney. The nurse then should notify the provider immediately. Skin turgor, lung sounds, and weight could give information about the fluid status of the client, but they are not the priority assessment.

B

A client is scheduled to have dialysis in 30 minutes and is due for the following medications: vitamin C, B-complex vitamin, and cimetidine (Tagamet). Which action by the nurse is best? a.Give medications with a small sip of water. b.Hold all medications until after dialysis. c.Give the supplements, but hold the Tagamet. d.Give the Tagamet, but hold the supplements. All three medications are dialyzable, meaning that they will be removed by the dialysis. They should be given after the treatment is over.

B

A nurse cares for a client who has elevated levels of antidiuretic hormone (ADH). Which disorder should the nurse identify as a trigger for the release of this hormone? a. Pneumonia b. Dehydration c. Renal failure d. Edema ADH increases tubular permeability to water, leading to absorption of more water into the capillaries. ADH is triggered by a rising extracellular fluid osmolarity, as occurs in dehydration. Pneumonia, renal failure, and edema would not trigger the release of ADH.

B

A nurse obtains a client's health history at a community health clinic. Which statement alerts the nurse to provide health teaching to this client? a. "I drink two glasses of red wine each week." b. "I take a lot of Tylenol for my arthritis pain." c. "I have a cousin who died of liver cancer." d. "I got a hepatitis vaccine before traveling." Acetaminophen (Tylenol) can cause liver damage if taken in large amounts. Clients should be taught not to exceed 4000 mg/day of acetaminophen. The nurse should teach the client about this limitation and should explore other drug options with the client to manage his or her arthritis pain. Two glasses of wine each week, a cousin with liver cancer, and the hepatitis vaccine do not place the client at risk for a liver disorder, and therefore do not require any health teaching.

B

A nurse plans care for a client with acute pancreatitis. Which intervention should the nurse include in this client's plan of care to reduce discomfort? a. Administer morphine sulfate intravenously every 4 hours as needed. b. Maintain nothing by mouth (NPO) and administer intravenous fluids. c. Provide small, frequent feedings with no concentrated sweets. d. Place the client in semi-Fowler's position with the head of bed elevated. The client should be kept NPO to reduce GI activity and reduce pancreatic enzyme production. IV fluids should be used to prevent dehydration. The client may need a nasogastric tube. Pain medications should be given around the clock and more frequently than every 4 to 6 hours. A fetal position with legs drawn up to the chest will promote comfort.

B

A preschool child is admitted to a psychiatric unit with the diagnosis autistic spectrum disorder. To help the child feel more secure on the unit, which intervention should a nurse include in this client's plan of care? A. Encourage and reward peer contact. B. Provide consistent caregivers. C. Provide a variety of safe daily activities. D. Maintain close physical contact throughout the day. Rationale: The nurse should provide consistent caregivers as part of the plan of care for a child diagnosed with autistic spectrum disorder. Children diagnosed with autistic spectrum disorder have an inability to trust. Providing consistent caregivers allows the client to develop trust and a sense of security.

B

A telehealth nurse speaks with a client who is recovering from a liver transplant 2 weeks ago. The client states, "I am experiencing right flank pain and have a temperature of 101° F." How should the nurse respond? a. "The anti-rejection drugs you are taking make you susceptible to infection." b. "You should go to the hospital immediately to have your new liver checked out." c. "You should take an additional dose of cyclosporine today." d. "Take acetaminophen (Tylenol) every 4 hours until you feel better." Fever, right quadrant or flank pain, and jaundice are signs of liver transplant rejection; the client should be admitted to the hospital as soon as possible for intervention. Anti-rejection drugs do make a client more susceptible to infection, but this client has signs of rejection, not infection. The nurse should not advise the client to take an additional dose of cyclosporine or acetaminophen as these medications will not treat the acute rejection.

B

After teaching a client who has alcohol-induced cirrhosis, a nurse assesses the client's understanding. Which statement made by the client indicates a need for additional teaching? a. "I cannot drink any alcohol at all anymore." b. "I need to avoid protein in my diet." c. "I should not take over-the-counter medications." d. "I should eat small, frequent, balanced meals." Based on the degree of liver involvement and decreased function, protein intake may have to be decreased. However, some protein is necessary for the synthesis of albumin and normal healing. The other statements indicate accurate understanding of self-care measures for this client.

B

Conduct disorder may be a precursor to the diagnosis of which personality disorder? a) Narcissistic personality disorder b) Antisocial personality disorder c) Histrionic personality disorder d) Passive-aggressive personality disorder

B

The nurse correlates which data in the client's history as a predisposing factor for Laënnec's cirrhosis? a. Gallstones b. Alcohol abuse c. Viral hepatitis d. Heart disease Laënnec's cirrhosis, also known as alcoholic cirrhosis, is caused by the toxic effect of alcohol on the liver. The nurse should ask the client about a history of alcohol use. The other factors are not related to this type of cirrhosis.

B

The nurse is assessing a client with a diagnosis of pre-renal acute kidney injury (AKI). Which condition would the nurse expect to find in the client's recent history? a. Pyelonephritis b. Myocardial infarction c. Bladder cancer d. Kidney stones Pre-renal causes of AKI are related to a decrease in perfusion, such as with a myocardial infarction. Pyelonephritis is an intrinsic or intrarenal cause of AKI related to kidney damage. Bladder cancer and kidney stones are post-renal causes of AKI related to urine flow obstruction.

B

The nurse is caring for a client who has undergone surgery to drain a pancreatic pseudocyst with placement of a pancreatic drainage tube. Which nursing intervention prevents complications from this procedure? a. Positioning the client in a right side-lying position b. Applying a skin barrier around the drainage tube site c. Clamping the drainage tube for 2 hours every 12 hours d. Irrigating the drainage tube daily with 30 mL of sterile normal saline The nurse assesses the skin around the drainage tube for redness or skin irritation, which can be severe from leakage of pancreatic enzymes. The nurse applies a skin barrier such as Stomahesive around the drainage tube to prevent excoriation. A side-lying position may be more comfortable for the client. The drainage tube should not be clamped or irrigated without specific orders.

B

The nurse is caring for four clients with chronic kidney disease. Which client should the nurse assess first upon initial rounding? a. Woman with a blood pressure of 158/90 mm Hg b. Client with Kussmaul respirations c. Man with skin itching from head to toe d. Client with halitosis and stomatitis Kussmaul respirations indicate a worsening of chronic kidney disease (CKD). The client is increasing the rate and depth of breathing to excrete carbon dioxide through the lungs. Hypertension is common in most clients with CKD, and skin itching increases with calcium-phosphate imbalances, another common finding in CKD. Uremia from CKD causes ammonia to be formed, resulting in the common findings of halitosis and stomatitis.

B

Which laboratory data does the nurse correlate with advanced disease in a client with cirrhosis? a.Elevated serum protein level b.Elevated serum ammonia level c.Decreased serum ammonia level d.Decreased lactate dehydrogenase level The serum ammonia level is elevated in the presence of advanced disease because conversion of ammonia to urea for excretion is decreased. The other laboratory values do not correlate with advanced disease.

B

A nurse assesses clients on the medical-surgical unit. Which clients are at risk for kidney problems? (Select all that apply.) a. A 24-year-old pregnant woman prescribed prenatal vitamins b. A 32-year-old bodybuilder taking synthetic creatine supplements c. A 56-year-old who is taking metformin for diabetes mellitus d. A 68-year-old taking high-dose nonsteroidal anti-inflammatory drugs (NSAIDs) for chronic back pain e. A 75-year-old with chronic obstructive pulmonary disease (COPD) who is prescribed an albuterol nebulizer Many medications can affect kidney function. Clients who take synthetic creatine supplements, metformin, and high-dose or long-term NSAIDs are at risk for kidney dysfunction. Prenatal vitamins and albuterol nebulizers do not place these clients at risk.

B, C, D

A nurse assesses a client who has a family history of polycystic kidney disease (PKD). For which clinical manifestations should the nurse assess? (Select all that apply.) a. Nocturia b. Flank pain c. Increased abdominal girth d. Dysuria e. Hematuria f. Diarrhea Clients with PKD experience abdominal distention that manifests as flank pain and increased abdominal girth. Bloody urine is also present with tissue damage secondary to PKD. Clients with PKD often experience constipation, but would not report nocturia or dysuria.

B, C, E

A nurse assesses a client who has liver disease. Which laboratory findings should the nurse recognize as potentially causing complications of this disorder? (Select all that apply.) a. Elevated aspartate transaminase b. Elevated international normalized ratio (INR) c. Decreased serum globulin levels d. Decreased serum alkaline phosphatase e. Elevated serum ammonia f. Elevated prothrombin time (PT) Elevated INR and PT are indications of clotting disturbances and alert the nurse to the increased possibility of hemorrhage. Elevated ammonia levels increase the client's confusion. The other values are abnormal and associated with liver disease but do not necessarily place the client at increased risk for complications.

B, E, F

How do you relive a patients pruritus?

Baking soda or Alpha Keri baths Lotions containing calamine; Antihistamines; Soft, old linen Control of the temperature (not too hot and not too cold).

Why does the Pancreas secrete Amylase?

Because it aids in the digestion of carbohydrates.

Why does the Pancreas secrete Lipase?

Because it aids in the digestion of carbohydrates.

Why does the Pancreas secrete Trypsin?

Because it aids in the digestion of carbohydrates.

Why is the Pancrease unusual?

Because it functions as both an exocrine gland and an endocrine gland.

A client has cirrhosis and has developed ascites and edema. Which laboratory value does the nurse correlate with this condition? a. Blood glucose, 120 mg/dL b. Serum sodium, 135 mEq/L c. Serum albumin, 2.1 g/dL d. Blood urea nitrogen, 18 mg/dL Ascites occurs as a result of the inability of the liver to synthesize albumin. Loss of albumin leads to edema. This client's albumin level is low, which correlates with the condition. Sodium and blood urea nitrogen (BUN) levels are normal. The glucose level is slightly high, but this is not directly related to edema.

C

A client has end-stage kidney disease (ESKD). Which food selection by the client demonstrates understanding of a low-sodium, low-potassium diet? a.Bananas b.Ham c.Herbs and spices d.Salt substitutes Herbs and spices can be used in place of salt to enhance food flavor. Bananas are high in potassium. Ham is high in sodium. Many salt substitutes contain potassium chloride and should not be used.

C

A client is diagnosed with chronic kidney disease (CKD). What is an ideal goal of treatment set by the nurse in the care plan to reduce the risk of pulmonary edema? a. Maintaining oxygen saturation of 89% b. Minimal crackles and wheezes in lung sounds c. Maintaining a balanced intake and output d. Limited shortness of breath upon exertion With an optimal fluid balance, the client will be more able to eject blood from the left ventricle without increased pressure in the left ventricle and pulmonary vessels. Other ideal goals are oxygen saturations greater than 92%, no auscultated crackles or wheezes, and no demonstrated shortness of breath.

C

A nurse assesses clients at a community health center. Which client is at highest risk for pancreatic cancer? a. A 32-year-old with hypothyroidism b. A 44-year-old with cholelithiasis c. A 50-year-old who has the BRCA2 gene mutation d. A 68-year-old who is of African-American ethnicity Mutations in both the BRCA2 and p16 genes increase the risk for developing pancreatic cancer in a small number of cases. The other factors do not appear to be linked to increased risk.

C

A nurse assesses clients on the medical-surgical unit. Which client is at greatest risk for the development of carcinoma of the liver? a. A 22-year-old with a history of blunt liver trauma b. A 48-year-old with a history of diabetes mellitus c. A 66-year-old who has a history of cirrhosis d. An 82-year-old who has chronic malnutrition The risk of contracting a primary carcinoma of the liver is higher in clients with cirrhosis from any cause. Blunt liver trauma, diabetes mellitus, and chronic malnutrition do not increase a person's risk for developing liver cancer.

C

A nurse prepares to assess the emotional state of a client with end-stage pancreatic cancer. Which action should the nurse take first? a. Bring the client to a quiet room for privacy. b. Pull up a chair and sit next to the client's bed. c. Determine whether the client feels like talking about his or her feelings. d. Review the health care provider's notes about the prognosis for the client. Before conducting an assessment about the client's feelings, the nurse should determine whether he or she is willing and able to talk about them. If the client is open to the conversation and his or her room is not appropriate, an alternative meeting space may be located. The nurse should be present for the client during this time, and pulling up a chair and sitting with the client indicates that presence. Because the nurse is assessing the client's response to a terminal diagnosis, it is not necessary to have detailed information about the projected prognosis; the nurse knows that the client is facing an end-of-life illness.

C

The nurse is assessing a client with mild liver disease. Which assessment does the nurse perform to detect the presence of ascites in this client? a.Measure lower extremities to assess for edema. b.Inspect and palpate the abdomen for distention. c.Palpate the abdomen in assessing for a fluid wave. d.Percuss the liver while listening for dullness. Mild ascites may be difficult to detect and can be assessed by percussion. Shifting dullness and a fluid wave alert the nurse to the presence of ascites. The other findings are inconsistent with ascites.

C

After teaching a client who is prescribed pancreatic enzyme replacement therapy, the nurse assesses the client's understanding. Which statement made by the client indicates a need for additional teaching? a. "The capsules can be opened and the powder sprinkled on applesauce if needed." b. "I will wipe my lips carefully after I drink the enzyme preparation." c. "The best time to take the enzymes is immediately after I have a meal or a snack." d. "I will not mix the enzyme powder with food or liquids that contain protein." The enzymes should be taken immediately before eating meals or snacks. If the client cannot swallow the capsules whole, they can be opened and the powder sprinkled on applesauce, mashed fruit, or rice cereal. The client should wipe his or her lips carefully after drinking the enzyme preparation because the liquid could damage the skin. Protein items will be dissolved by the enzymes if they are mixed together.

C

The nurse is caring for a client who is receiving peritoneal dialysis (PD). Which nursing intervention has the greatest priority when a dialysis exchange is performed? a.Adding potassium and antibiotic to the dialysate bags b.Positioning the client on either side c.Using sterile technique when hooking up dialysate bags d.Warming the dialysate fluid in a microwave oven Peritonitis is the major complication of PD. The most common cause of peritonitis is connection site contamination. To prevent peritonitis, use meticulous sterile technique when caring for the PD catheter and when hooking up or clamping off dialysate bags. This safety precaution is the priority. Never warm dialysate fluid in the microwave. Positioning the client may help with the flow of fluid. Clients may need additives to their dialysate fluid, but potassium and antibiotics are not added together because interactions between them can reduce the effectiveness of the antibiotic.

C

The nurse is taking the vital signs of a client after hemodialysis. Blood pressure is 110/58 mm Hg, pulse 66 beats/min, and temperature is 99.8° F (37.6° C). What is the most appropriate action by the nurse? a. Administer fluid to increase blood pressure. b. Check the white blood cell count. c. Monitor the client's temperature. d. Connect the client to an electrocardiographic (ECG) monitor. During hemodialysis, the dialysate is warmed to increase diffusion and prevent hypothermia. The client's temperature could reflect the temperature of the dialysate. There is no indication to check the white blood cell count or connect the client to an ECG monitor. The other vital signs are within normal limits.

C

The nurse is teaching a client how to increase the flow of dialysate into the peritoneal cavity during dialysis. Which statement by the client demonstrates a correct understanding of the teaching? a. "I should leave the drainage bag above the level of my abdomen." b. "I could flush the tubing with normal saline if the flow stops." c. "I should take a stool softener every morning to avoid constipation." d. "My diet should have low fiber in it to prevent any irritation." Inflow and outflow problems of the dialysate are best controlled by preventing constipation. A daily stool softener is the best option for the client. The drainage bag should be below the level of the abdomen. Flushing the tubing will not help with the flow. A diet high in fiber will also help with a constipation problem.

C

Which should be the priority nursing intervention when caring for a child diagnosed with conduct disorder? A. Modify environment to decrease stimulation and provide opportunities for quiet reflection. B. Convey unconditional acceptance and positive regard. C. Recognize escalating aggressive behavior and intervene before violence occurs. D. Provide immediate positive feedback for appropriate behaviors. Rationale: The priority nursing intervention when caring for a child diagnosed with conduct disorder should be to recognize escalating aggressive behavior and to intervene before violence occurs. This intervention serves to keep the client as well as others safe, which is the priority nursing concern.

C

Which situation reflects violation of the ethical principle of veracity? A. A nurse discusses with a client another client's impending discharge. B. A nurse refuses to give information to a physician who is not responsible for the client's care. C. A nurse tricks a client into seclusion by asking the client to carry linen to the seclusion room. D. A nurse does not treat all of the clients equally, regardless of illness severity. Rationale: The nurse who tricks a client into seclusion has violated the ethical principle of veracity. The principle of veracity refers to one's duty to always be truthful and not intentionally deceive or mislead clients.

C

Which statement made by a client traveling to a nonindustrialized country indicates the need for further teaching regarding the prevention of viral hepatitis? a."I will drink bottled water while I'm gone." b."I will not share my drinking glass." c."I should eat plenty of fresh fruits and vegetables." d."I will use careful handwashing." The client should be advised to avoid fresh, raw fruits and vegetables because they can be contaminated by tap water. Drinking bottled water, not sharing glasses (or eating utensils), and careful handwashing are all good ways to prevent illness.

C

Which foods will the nurse teach the client with chronic pancreatitis to avoid? Select all that apply. A. Blueberries B. Green beans C. Bacon D. Baked fish E. Fried potatoes

C,E

What are the 2 types of Conduct Disorder?

Childhood onset & adolescent onset.

Laennec's or alcoholic cirrhosis is caused by?

Chronic alcoholism

What is the most common causes of cirrhosis?

Chronic alcoholism, chronic viral hepatitis, nonalcoholic steatohepatitis (NASH), bile duct disease, and genetic diseases

Biliary cirrhosis also called cholestatic; caused by?

Chronic biliary obstruction or autoimmune disease

What is the most common reason for liver transplantation?

Cirrhosis, chronic hepatitis B and hepatitis C, bile duct diseases, autoimmune liver disease, primary liver cancer, alcoholic liver disease, and fatty liver disease.

For the patient who has undergone liver transplantation, monitor for what?

Clinical manifestations of rejection, which may include tachycardia, fever, right upper quadrant or flank pain, decreased bile pigment and volume, and increasing jaundice.

Recovery Phase

Continued improvement of uremic symptoms Decreasing BUN and creatinine Acid-base, fluid and electrolyte balance Return to normal activity level Kidney's decreased ability to concentrate urine may be mild and continue for years

A client has just had a central line catheter placed that is specific for hemodialysis. What is the most appropriate action by the nurse? a. Use the catheter for the next laboratory blood draw. b. Monitor the central venous pressure through this line. c. Access the line for the next intravenous medication. d. Place a heparin or heparin/saline dwell after hemodialysis. The central line should have a heparin or heparin/saline dwell after hemodialysis treatment. The central line catheter used for dialysis should not be used for blood sampling, monitoring central venous pressures, or giving drugs or fluids.

D

A client with acute cholecystitis is admitted to the medical-surgical unit. Which nursing activity associated with the client's care will be BEST for the nurse to delegate to unlicensed assistive personnel (UAP)? A. Assessing dietary risk factors for cholecystitis B. Checking for bowel sounds and distention C. Determining precipitating factors for abdominal pain D. Obtaining the admission weight, height, and vital signs Obtaining height, weight, and vital signs is included in the education for UAP and usually is included in the job description for these staff members. Assessment, checking bowel sounds, and determining precipitating factors for abdominal pain require broader education and are within the scope of practice of licensed nursing staff.

D

A client with chronic kidney disease states, "I feel chained to the hemodialysis machine." What is the nurse's best response to the client's statement? a. "That feeling will gradually go away as you get used to the treatment." b. "You probably need to see a psychiatrist to see if you are depressed." c. "Do you need help from social services to discuss financial aid?" d. "Tell me more about your feelings regarding hemodialysis treatment." The nurse needs to explore the client's feelings in order to help the client cope and enter a phase of acceptance or resignation. It is common for clients to be discouraged because of the dependency of the treatment, especially during the first year. Referrals to a mental health provider or social services are possibilities, but only after exploring the client's feelings first. Telling the client his or her feelings will go away is dismissive of the client's concerns.

D

A nurse cares for a client who is hemorrhaging from bleeding esophageal varices and has an esophagogastric tube. Which action should the nurse take first? a. Sedate the client to prevent tube dislodgement. b. Maintain balloon pressure at 15 and 20 mm Hg. c. Irrigate the gastric lumen with normal saline. d. Assess the client for airway patency. Maintaining airway patency is the primary nursing intervention for this client. The nurse suctions oral secretions to prevent aspiration and occlusion of the airway. The client usually is intubated and mechanically ventilated during this treatment. The client should be sedated, balloon pressure should be maintained between 15 and 20 mm Hg, and the lumen can be irrigated with saline or tap water. However, these are not a higher priority than airway patency.

D

What are the 5 stages of Hepatic Encephalopathy?

Grade 0- insomnia, sleep disturbances Grade 1-Lack of awareness; personality change Grade 2- Lethargy, drowsiness, inappropriate behavior Grade 3-Asleep, but arousable Grade 4-Not arousable

What are the medications approved for Tourette's Syndrome?

Haloperidol (Haldol) Pimozide (Orap) Risperidone (Risperdal) Olanzapine (zyprexa) Ziprasidone (Geodon)

After studying the DSM-5 criteria for oppositional defiant disorder (ODD), which listed symptom would a student nurse recognize? A. Arguing and annoying older sibling over the past year B. Angry and resentful behavior over a three-month period C. Initiating physical fights for more than 18 months D. Arguing with authority figures for more than six months Rationale: The DSM-5 rules out the diagnosis of ODD when only siblings are involved in argumentative interactions. Angry and resentful behavior over more than six months, not three months, would be considered a symptom of ODD. Initiating physical fights is a symptom of conduct disorder, not ODD. Arguing with authority figures for more than six months is listed by the DSM-5 as a symptom for the diagnosis of ODD.

D

The nurse is caring for a client recently diagnosed with type 1 diabetes mellitus who has had an episode of acute pancreatitis. The client asks the nurse how he developed diabetes when the disease does not run in the family. What is the nurse's BEST response? A. "The diabetes could be related to your obesity." B. "What has your doctor told you about your disease?" C. "Do you consume alcohol on a frequent basis?" D. "Type 1 diabetes can occur when the pancreas is destroyed by disease." Telling the client that type 1 diabetes can occur when the pancreas is destroyed by disease is the only response that accurately describes the relationship of the client's diabetes to pancreatic destruction. Type 2, not type 1, diabetes is usually related to obesity. Asking the client what the provider has said is an evasive response by the nurse and does not address the client's question. Many factors could produce acute pancreatitis other than alcohol consumption.

D

The nurse is caring for a client with chronic kidney disease who has developed uremia. Which assessment finding does the nurse correlate with this problem? a.Decreased breath sounds b.Foul-smelling urine c.Heart rate of 50 beats/min d.Respiratory rate of 40 breaths/min A client with uremia will also have metabolic acidosis. With severe metabolic acidosis, the client will develop hyperventilation, or Kussmaul respirations, as the body attempts to compensate for the falling pH. The other manifestations would not be associated with acidosis.

D

The nurse is providing teaching for a client scheduled for a paracentesis. Which statement by the client indicates the teaching has been successful? A. "I must not use the bathroom prior to the procedure." B. "I will lie on my stomach while the procedure is performed." C. "I will not be allowed to eat or drink anything the night before surgery." D. "The physician will likely remove 2 to 3 liters of fluid from my abdomen."

D

The nurse is providing teaching for a client scheduled for a paracentesis. Which statement by the client indicates the teaching has been successful? A. "I must not use the bathroom prior to the procedure." B. "I will lie on my stomach while the procedure is performed." C. "I will not be allowed to eat or drink anything the night before surgery." D. "The physician will likely remove 2 to 3 liters of fluid from my abdomen." Rationale: The client should void before the procedure to prevent injury to the bladder. The clientwill lie in bed with the head of the bed elevated during the procedure

D

Which intervention is most important for the nurse to implement in a client after kidney transplant surgery? a.Promote acceptance of new body image. b.Monitor magnesium levels daily. c.Place the client on protective isolation. d.Remove the indwelling (Foley) catheter as soon as possible. Because of increased risk for infection related to immune suppressive drugs given to prevent rejection, the catheter is removed as soon as possible to avoid infection, usually 3 to 7 days after surgery. The client may need assistance with changes in body image, but this is not the priority. The client does not require protective precautions. Laboratory values will be monitored frequently in a post-transplant client, but this is not as important as preventing a complication by removing the catheter.

D

Which is a potential side effect from the prolonged use of methylphenidate (Ritalin)? a) Psychosis b) A decreased intelligence quotient (IQ) c) Weight gain d) A decrease in rate of growth and development

D

Which set of assessment findings indicates to the nurse that a client may have acute pancreatitis? A. Absence of jaundice, pain of gradual onset B. Absence of jaundice, pain in right abdominal quadrant C. Presence of jaundice, pain worsening when sitting up D. Presence of jaundice, pain worsening when lying supine Pain that worsens when lying supine and the presence of jaundice are the only assessment findings indicative of acute pancreatitis. Pain associated with acute pancreatitis usually has an abrupt onset, is located in the mid-epigastric or upper left quadrant, and lessens with sitting up; also, jaundice is present.

D

What behavior can cause Pancreatitis?

Drinking alcohol or eating a high-fat meal.

What are the side effects of Risperidone (Risperdal)?

Drowsiness, increased appetite, nasal congestion, fatigue, constipation, drooling, dizziness, weight gain.

A branch of philosophy that addresses methods for determining the rightness or wrongness of one's actions is defined as _______________________. Rationale: Ethics is a branch of philosophy that deals with systematic approaches to distinguishing right from wrong behavior. Bioethics is the term applied to these principles when they refer to concepts within the scope of medicine, nursing, and allied health.

ETHICS

Vital signs for Acute Pancreatitis to look for?

Elevated temperature, tachycardia, and decreased blood pressure, Respiratory problems, such as left lung pleural effusions, atelectasis, and pneumonia, are common in patients with acute pancreatitis. Auscultate the lung fields for adventitious sounds or diminished breath sounds, and observe for dyspnea or orthopnea. Obtain the patient's weight, He or she may report weight loss resulting from nausea and vomiting.

What are the 2 functions of the Pancreas?

Endocrine & Exocrine

What is the hallmark of pancreatic necrosis?

Enzymatic fat necrosis of the endocrine and exocrine cells of the pancreas caused by the enzyme lipase.

What is the primary risk factor for a patient developing Acute Pancreatitis?

Excessive alcohol ingestion.

What are the two categories of Acute Pancreatitis?

Hemorrhagic & Non-hemorrhagic

chronic liver disease transmitted via blood chronic liver disease occurs in 85% of infected treat with Ribavirin and a- interferon most don't recover

Hepatitis C

What causes Jaundice? (yellowish coloration of the skin)

Hepatocellular disease or intrahepatic obstruction.Patients with jaundice often report pruritus (itching).

What are some things that would be important to ask a client that might have Cirrhosis?

History of alcoholism Viral Hepatitis Chronic biliary disease Medication history including over the counter meds Cardiac problems GI problems including bleeding Black tarry stools Spider veins Petechiae

How do you treat mild pancreatitis?

Hydration with IV fluids, pain control, and drug therapy, fasting and rest can also relive pain.

What are Clinical Signs/Symptoms of Pancreatic Cancer?

Hyperglycemia, GI Bleeding, Leg or calf pain (thrombophlebitis), Fatigue, weakness, Splenomegaly

What kind of IV fluids would the DR prescribe?

IV isotonic fluid administration to maintain hydration, IV of calcium and magnesium may also be needed.

How do you treat Hyperacute Rejection?

Immediate removal of the transplanted kidney

What are the Clinical Manifestations of Hyperacute Rejection?

Increased temperature Increased blood pressure Pain at transplant site

What can an elevated Serum Amylase level mean?

Indicate possible Acute pancreatitis.

DKA Treatment

Insulin: Plasma glucose should fall @ least 10% per hour if adequate insulin is given Insulin Protocol: IV bolus of 5-10 units Regular Insulin followed by continuous regular insulin drip (0.1 U/kg/hr) via IV pump until BS reached 250. Infusion is then stopped. A dose of SC Insulin will be given 30-60 minutes prior to stopping of drip Repeat SC Insulin doses PRN

Most deaths in patients with acute pancreatitis result from what?

Irreversible shock

How does a Balloon Tamponade Sengstaken-Blakemore work?

It controls hemorrhage by compression of varices (gastric or esophageal)

What is Hepatitis A?

It is a hardy virus and survives on human hands. The virus is resistant to detergents and acids but is destroyed by chlorine (bleach) and extremely high temperatures.It has a mild course similar to that of a typical flu-like INFECTION and often goes unrecognized.

What is Transjugular intrahepatic portosystemic shunt (TIPS)?

It is a procedure to create new connections between two blood vessels in your liver. Used to redirect portal blood flow & decrease portal venous pressure and decompresses the varices.

Why should you keep a pair scissors the bed for balloon tamponade?

It is extremely important to monitor the patient continuously, as the tube can migrate and occlude the airway. Keep a pair of scissors at the bedside so that if this does occur, the tube can be cut immediately directly below the bifurcation and removed.

What is Hepatitis B?

It is not transmitted like Hepatitis A. It is a double-shelled particle containing DNA composed of a core antigen (HBcAg), a surface antigen (HBsAg), and another antigen found within the core (HBeAg) that circulates in the blood.

How is Hepatitis A spread?

It is spread most often by the fecal-oral route by fecal contamination either from person-to-person contact (e.g., oral-anal sexual activity) or by consuming contaminated food or water. Common sources of infection include shellfish caught in contaminated water and food contaminated by food handlers infected with Hepatitis A.

What is Hepatitis?

It is the widespread INFLAMMATION of liver cells.

What is Lactulose (Cephulac) used to treat?

It is used to expel the ammonia from patients that have Hepatic Encephalopathy, the laxative in it causes diarrhea which is an expected effect.

What is fetor hepaticus?

Liver breath.

What is Thrombocytopenia?

Low serum platelet count.

What is the exocrine function of the Pancreas?

Manages digestive processes through the release of pancreatic enzymes necessary for digestion and absorption to occur of starches, proteins and fats.

How is Hepatitis C spread?

Most commonly by: • Illicit IV drug needle sharing (highest incidence) • Blood, blood products, or organ transplants received before 1992 • Needle stick injury with HCV-contaminated blood (health care workers at high risk) • Unsanitary tattoo equipment • Sharing of intranasal cocaine paraphernalia The disease is not transmitted by casual contact or by intimate household contact. However, those infected are advised not to share razors, toothbrushes, or pierced earrings because microscopic blood may be on these items.

What are the characteristics of Tourette's Syndrome?

Motor tics and one or more vocal tics. May occur simultaneously or at different periods. Can cause distress and interfere w/ occupational or social function. Onset common at 6 or 7. More common in boys. Lifelong or diminish in adulthood

What is Hemorrhagic Pancreatitis?

Necrotic tissue damage extends to vascular compartments causing diffuse bleeding, fibrosis, tissue death.

Characteristics of DKA?

Occurs with severe insulin deficiency Type I DM Occurs in undiagnosed & diagnosed diabetics (infection, trauma, non-compliance, stressors, insulin not taken as prescribed) Change in diet, exercise Gradual onset Severe Hyperglycemia Ketosis- blood and urine ketones Acidosis Severe Dehydration Severe Fluid/Electrolyte Imbalance

What are the symptoms of Steel Syndrome?

Pallor Diminished pulses (distal to the fistula) Necrosis[1] Decreased wrist-brachial index (ratio of blood pressure measured in the wrist and the blood pressure measured in the upper arm)

The nurse closely monitors the client with acute pancreatitis for which complication?

Pneumonia

What are the predisposing factors of Oppositional Defiant Disorder (ODD)?

Power and control are issues for parents or if they exercise authority for their own needs a power struggle can be established between the parent and child setting the stage for ODD. Downward spiral when parent try to control the child. Rewarding the behavior w/ the attention given even if it is negative

Typically, a patient is diagnosed with Pancreatitis after?

Presenting with severe abdominal PAIN in the mid-epigastric area or left upper quadrant. The patient often states that the pain had a sudden onset and radiates to the back, left flank, or left shoulder. The pain is described as intense, boring (feeling that it is going through the body), and continuous.

What is the endocrine function of the Pancreas?

Produces several important hormones such as insulin, glucagon, and somatostatin. These compounds and many more play a vital role in lipid and carbohydrate metabolism

What is Hepatorenal syndrome?

Serious complication of cirrhosis and liver decompensation, systemic vasodilation, decreased arterial blood volume cause renal vasoconstriction, decrease renal perfusion - Renal Failure

What are the ANA Code of Ethics?

Provision 1 The nurse practices with compassion and respect for the inherent dignity, worth, and unique attributes of every person. Provision 2 The nurse's primary commitment is to the patient, whether an individual, family, group, community, or population. Provision 3 The nurse promotes, advocates for, and protects the rights, health, and safety of the patient. Provision 4 The nurse has authority, accountability, and responsibility for nursing practice; makes decisions; and takes action consistent with the obligation to promote health and to provide optimal care. Provision 5 The nurse owes the same duties to self as to others, including the responsibility to promote health and safety, preserve wholeness of character and integrity, maintain competence, and continue personal and professional growth. Provision 6 The nurse, through individual and collective effort, establishes, maintains, and improves the ethical environment of the work setting and conditions of employment that are conducive to safe, quality health care. Provision 7 The nurse, in all roles and settings, advances the profession through research and scholarly inquiry, professional standards development, and the generation of both nursing and health policy. Provision 8 The nurse collaborates with other health professionals and the public to protect human rights, promote health diplomacy, and reduce health disparities. Provision 9 The profession of nursing, collectively through its professional organizations, must articulate nursing values, maintain the integrity of the profession, and integrate principle of social justice into nursing and health policy.

A valid, legally recognized claim or entitlement, encompassing both freedom from government interference or discriminatory treatment and an entitlement to a benefit or a service is defined as a _______________________. Rationale: A right is a valid, legally recognized claim or entitlement, encompassing both freedom from government interference or discriminatory treatment and an entitlement to a benefit or a service. A right is absolute when there is no restriction whatsoever on the individual's entitlement.

RIGHT

The growth rate of Pancreatic Cancer is?

Rapid growth and is metastasize to surrounding organs.

What are the Hepatitis A vaccines called?

Recombivax-HB & Engerix-B

Why is Bleeding esophageal varices is a life-threatening medical emergency?

Severe blood loss may occur, resulting in shock from hypovolemia. The bleeding may be either hematemesis (vomiting blood) or melena (black, tarry stools). Loss of consciousness may occur before any observed bleeding. Variceal bleeding can occur spontaneously with no precipitating factors.

How do you diagnose Acute Pancreatitis?

Severe mid-epigastric pain Serum amylase elevated Transient hyperglycemia Decreased bowel sounds upper gastrointestinal (UGI) series Liver function studies abnormal

What is the number one clinical symptom of Acute Pancreatitis in a patient?

Severe pain, worse than cancer.

How is Hepatitis B spread?

Spread through these common modes of transmission: • Unprotected sexual intercourse with an infected partner • Sharing needles • Accidental needle sticks or injuries from sharp instruments primarily in health care workers (low incidence) • Blood transfusions (that have not been screened for the virus, before 1992) • Hemodialysis • Close person-to-person contact by open cuts and sores

How do you treat Acute Pancreatitis?

Stabilization and shock reversal Pain management Management of metabolic issues Minimize pancreatic stimulation

Nursing Management of Acute Pancreatitis?

Supportive care to relieve symptoms, prevent/treat complications Pain management- IV narcotic analgesia NPO - rest the pancreas and reduce enzyme secretion IV infusion of isotonic fluid hydration NG tube for gastric decompression and suppress pancreatic secretion.

What is Non-hemorrhagic Pancreatitis?

The Pancreas is edematous; potent enzymes escape to surrounding tissues and the gland swells.

What is a Pseudocyst?

The cavity surrounding pancreas gets filled with necrotic products.

The severity of pancreatitis depends on what?

The extent of inflammation and tissue damage.

What body position can a patient exprecing Acute Pancreatitus can ease the pain?

The fetal position (with the knees drawn up to the chest and the spine flexed) or by sitting upright and bending forward. He or she may report weight loss resulting from nausea and vomiting. Obtain the patient's weight.

How do you treat HSS?

The main goal of HHS treatment includes correcting dehydration which would improve blood pressure, circulation, and urine output.

What are Esophageal varices?

The result of portal hypertension, the blood backs up from the liver and enters the esophageal and gastric veins. They occur when fragile, thin-walled esophageal veins become distended and tortuous from increased pressure.

Because paralytic (adynamic) ileus is a common complication of acute pancreatitis, prolonged nasogastric intubation may be necessary. What should you asses for?

The return of peristalsis by asking the patient if he or she has passed flatus or had a stool. The return of bowel sounds is not reliable as an indicator of peristalsis return; passage of flatus or a bowel movement is the most reliable indicator

What position would a patient report hurts more in Acute Pancreatitis?

The supine position.

What specific blood tests diagnose pancreatic cancer?

There are none.

What are Kussmaul respirations?

They are very deep and rapid respirations causing respiratory alkalosis to correct metabolic acidosis by exhaling carbon dioxide.

What does DIABETIC KETOACIDOSIS (DKA) have in common with HYPERGLYCEMIC-HYPEROSMOLAR STATE (HHS)?

They both can be brought on by an infection, or other stressors. They both can cause dehydration and electrolyte imbalances. Both DKA and HHS are the two complications of diabetes mellitus.

What is the first clinical sign that a patient has liver dysfunction?

Thrombocytopenia

To rest the pancreas and reduce pancreatic enzyme secretion a nurse should do what?

Withhold food and fluids (NPO) during the acute period.

Can What is Hepatorenal syndrome be reversed?

Yes, but only with a liver transplant.

Is the diseased liver removed during a liver transplant?

Yes, it is.

Can Hepatic Encephalopathy be reversed?

Yes, with early intervention. Later neurologic symptoms include an altered level of consciousness, impaired thinking processes, and neuromuscular problems.

A client with an acute kidney injury will require continuing dialysis even after discharge. What does the nurse include in the discharge teaching? A Consume foods rich in potassium. B Limit the intake of foods containing phosphorus. C Consume at least 2 L of fluids daily. D Report a daily weight gain of more than 3 lb. This client should limit the intake of foods containing phosphorus because high blood phosphate levels cause hypocalcemia. The client should restrict dietary sodium and potassium; low urine output can cause an increase in serum sodium and serum potassium levels. Fluid consumption is restricted in the client to prevent fluid overload. The client must keep a log of daily weight measurements and report a weight gain of 2 lb a day to the health care provider.

b

What meds should you not give prior to dialysis?

antihypertensives, water soluble vitamins

What is Steel Syndrome?

blockage of the fisula

What is the danger of untreated disequilibrium syndrome?

cerebral edema. increased inter cranial pressure, coma, possible death

Hepatitis A

fecal oral route /react-text most common /react-text self limiting

What is disequilibrium syndrome?

happens within the first hour, arises in individuals starting hemodialysis due to chronic renal failure and is associated, in particular, with "aggressive" (high solute removal) dialysis. However, it may also arise in fast onset, i.e. acute, renal failure in certain conditions.

What are the Signs & Symptoms of disequilibrium syndrome?

headaches, nausea , vomiting, restlessness, decrease levels of consciousness, seizures

What medical problems can cause Pancreatitis?

including peptic ulcer disease, renal failure, vascular disorders, hyperparathyroidism, and hyperlipidemia, also inquire about recent viral infections.

Why do we warm up the Dialysate?

increases the efficiency of the diffusion process, we don't want to lower the patients body temperature

What are the hormones secreted by the Pancreas?

insulin, glucagon, and somatostatin

What is the target organ for ADH?

kidneys

What are the predisposing factors for Autism Spectrum Disorder (ASD)?

• Abnormalities in brain structure or function. • Certain medical conditions (tuberous sclerosis, fragile x syndrome, maternal rubella) • Familial association, chromosomal involvement (test parents) • Maternal asthma or allergies

Signs & Symptoms of Hepatitis B are what?

• Anorexia, nausea, and vomiting • Fever • Fatigue • Right upper quadrant pain • Dark urine with light stool • Joint PAIN • Jaundice

What is the long-term management of Varicies?

•β-adrenergic blockers/Propranolol •Repeated sclerotherapy/band ligation •Portosystemic shunts

What is Cirrhosis?

It is extensive, irreversible scarring of the liver, usually caused by a chronic reaction to hepatic INFLAMMATION and necrosis. The disease typically develops slowly and has a progressive, prolonged, destructive course resulting in end-stage liver disease.

What causes Intrahepatic obstructive jaundice?

Results from edema, fibrosis, or scarring of the hepatic bile channels and bile ducts, which interferes with normal bile and bilirubin excretion. Patients with jaundice often report pruritus (itching).

What medications are approved for Autism Spectrum Disorder (ASD)?

Risperidone & Aripiprazole

What are the side effects of Aripiprazole (Abilify)?

Sedation, fatigue, weight gain, vomiting, somnolence, tremor

What are the levels of Serum amylase, lipase levels and bilirubin like in Pancreatic Cancer?

They are elevated.

What are the characteristics of Pancreatic tumors?

They are highly malignant, usually adenocarcinomas and grow in well-defined, glandular patterns.

What does Obtund mean?

To dull or blunt. Often used in reference to diminished consciousness.

Why should you Deflate q8-12 hrs. a Balloon Tamponade Sengstaken-Blakemore?

To prevent necrosis of the lining of the esophagus.

Why would you use Zantac or Protonix to treat Acute Pancreatitis?

To reduce gastric acid secretion.

What is a Hepatitis A & B vaccine called?

Twinrix

Diuretic Phase

Urine production occurs Initial urine output of 1-3 liters/day and may increase to 3-5 liters/day

Oliguric Phase

Urine production of < 400 ml in 24 hours

Why is PROJECTION experienced by Oppositional Defiant Disorder (ODD)?

Usually they do not see themselves as being oppositional, but view the problem as result of others making unreasonable demands on them.

What is the target organ for Oxytocin?

Uterus and mammary glands.

What is a common complication of pancreatic cancer?

Venous thromboembolism

Postnecrotic cirrhosis is caused by?

Viral hepatitis [especially hepatitis C] and certain drugs or other toxins (caused by viral hepatitis [especially hepatitis C] and certain drugs or other toxins)

When is Hepatitis considered chronic?

When liver inflammation lasts longer than 6 months.

What meds should you give prior to dialysis?

regular insulin

Negative feedback prevents?

release

Positive feedback stimulates what?

secretion

What is a dry weight?

taken after dialysis, after fluid taken off

The oliguric phase of acute kidney failure is characterized by ?

the accumulation of nitrogenous wastes, resulting in increasing levels of serum creatinine and potassium, bicarbonate deficit, and decreased or no urine output

What is Protamine sulfate?

the antidote to heparin

What is a thrill?

the feel of an AV fistula

What is a bruit?

the swishing sound of an AV fistula

What is Allen's test?

to determine whether the patency of the radial or ulnar artery is normal

Hepatitis B

transmitted via blood hbv- sexual contact needles, tattoos, hemodialysis can live on dry surface for one week vaccines for this

What are the characteristics of Conduct Disorder?

use of physical aggression to violate rights of others bully's, threatens, intimidates others w/ weapon use, cruel to people and animals, repetitive and persistent pattern of behavior in which the basic rights of others and major age appropriate rules and societal norms are violated. Stealing, lying, truancy, fire, destruction of property, cons others, stays out, runs away, uses PROJECTION

How do you treat disequilibrium syndrome?

with IV hypertonic fluids, such as D5W with ringers lactate, D5, any hypertonic solutions

When would Hyperacute Rejection happen?

within the first 48 hours

What are the Signs & Symptoms of hepatorenal syndrome (HRS)?

• A sudden decrease in urinary flow (<500 mL/24 hr) (oliguria) • Elevated blood urea nitrogen (BUN) and creatinine levels with abnormally decreased urine sodium excretion • Increased urine osmolarity HRS often occurs after clinical deterioration from GI bleeding or the onset of hepatic encephalopathy.

What are the signs and symptoms of DKA?

• Can happen rapidly •mortality rate 2 to 5 per cent •mostly type 1, sometimes type 2 • hyperglycemia • dehydration • ketoacidosis • increased thirst and • increased urination • malaise, • weakness • fatigue.

What are the signs and symptoms of HSS?

• Can take as long as a week to develop • higher mortality rate ,15 percent. • is mainly seen in older patients having type 2 diabetes. • Increased dehydration, acute illness, vomiting, dementia, pneumonia, immobility, and UTI's

What are the 3 glucose-related emergencies can occur in patients with diabetes?

• Diabetic ketoacidosis (DKA) caused by lack of insulin and ketosis • Hyperglycemic-hyperosmolar state (HHS) caused by insulin deficiency and profound dehydration • Hypoglycemia from too much insulin or too little glucose

What are the characteristics of ADHD?

• Difficulty performing age appropriate tasks, highly distractible, extremely limited attention span, impulsive • Difficulty forming satisfactory interpersonal relationships, demonstrates behavior that inhibit acceptable social interaction • Disruptive and intrusive in group endeavors (always leaving seat) • Excessive levels of activity, restlessness, and fidgeting • Low frustration tolerance and temper outbursts • Running, jumping, wiggling, squirming

Signs & Symptoms of Pancreatic Cancer include what?

• Jaundice • Clay-colored (light) stools • Dark urine • Abdominal pain: usually vague, dull, or nonspecific that radiates into the back • Weight loss • Anorexia • Nausea or vomiting • Glucose intolerance • Splenomegaly (enlarged spleen) • Flatulence • Gastrointestinal bleeding • Ascites (abdominal fluid) • Leg or calf pain (from thrombophlebitis) • Weakness and fatigue

How can you avoid Hepatitis A?

• Proper handwashing, especially after handling shellfish • Avoiding contaminated food or water (including tap water in countries with high incidence) • Receiving immunoglobulin within 14 days if exposed to the virus • Receiving the HAV vaccine before traveling to areas where the disease is common (e.g., Mexico, Caribbean) • Receiving the vaccine if living or working in enclosed areas with others, such as college dormitories, correctional institutions, day-care centers, and long-term care facilities Several HAV vaccines are available (e.g., Havrix and Vaqta). Both of these vaccines are made of inactivated hepatitis A virus and are given in the deltoid muscle.

Patients who are not considered candidates for transplantation are those with?

• Severe cardiovascular instability with advanced cardiac disease • Severe respiratory disease • Metastatic tumors • Inability to follow instructions regarding drug therapy and self-management

Care for the patient that has had a a Balloon Tamponade

• Verify placement by xray • Prevent coughing • Deflate q8-12 hrs. to prevent necrosis • Most common complication Aspiration pneumonia (PNA) • Keep Scissors at bedside to cut tube

What are the predisposing factors to Tourette's?

• complication of pregnancy, • Low birth weight • head trauma, • CO poisoning • encephalitis

Cirrhosis abnormal Lab Findings include what?

•Anemia •Thrombocytopenia •Low albumin •Low potassium •Abnormal Liver Function Tests (LFTS) •Increased PT/PTT, Ammonia and Bilirubin •Increased Creatinine

What is Spontaneous Bacterial Peritonitis?

•Bacterial infection of ascitic fluid, causes alterations in immune function •Occurs in 8-30% of hospitalized pts with Cirrhosis and ascites and common after variceal hemorrhage

What are the Signs & Symptoms of Hepatic Encephalopathy?

•Change in mental status or Responsiveness •Increased Ammonia level •Asterixis - Flapping tremors •Fetor hepaticus- Musty, sweet odor on pts. Breath

What is drug therapy for Esophageal/Gastric Varices?

•Octreotide (Sandostatin) •Vasopressin (VP, Terlipressin) •Nitroglycerin (NTG) •β-adrenergic blockers (Inderal)

What are the characteristics of Autism Spectrum Disorder (ASD)?

•Withdrawal of child into the self and into a fantasy world of their own creation. •Aversion to affection and physical contact • Verbal/ Non-verbal skills affected, language is absent or has immature structure, and facial expressions or gestures are absent or socially inappropriate. • imaginative play is restricted, only exists internally w/ self. • Parents may first think the child is deaf • Resists change or becomes hysterical w/ change. Obsessed w/ routines • Likes spinning objects (fans) • Stereotyped body movements • Abnormal diet (lots of fluid, limited things that they like) • SIB (banging head)

What are the five major predisposing factors of IDD?

•hereditary factors • early alterations in embryonic development • pregnancy and perinatal factors • medical conditions acquired in infancy or childhood • environmental influences and other mental disorders.

A client has just been diagnosed with pancreatic cancer. The client's upset spouse tells the nurse that they have recently moved to the area, have no close relatives, and are not yet affiliated with a church. What is the nurse's BEST response? A. "Maybe you should find a support group to join." B. "Would you like me to contact the hospital chaplain for you?" C. "Do you want me to try to find a therapist for you?" D. "Do you have any friends whom you want me to call?" It is appropriate for the nurse to suggest contacting the hospital chaplain as a counseling option for the client and family. Suggesting that the client find a support group does not assist the client and the family with the problem. It is inappropriate for the nurse to suggest that the client and the family need a therapist. The spouse has already told the nurse that they have recently moved to the area, so it is unlikely that they have already made close friends.

B

The DSM-5 criteria for ODD specifies that: A persistent pattern of angry/irritable mood, argumentative/defiant behavior, or vindictiveness must be evident and last at least ______________ months. Rationale: A persistent pattern of angry/irritable mood, argumentative/defiant behavior, or vindictiveness must be evident and last at least six months according to the DSM-5 criteria for the diagnosis of ODD.

6 MONTHS

A 70-kg adult with chronic renal failure is on a 40-g protein diet. The client has a reduced glomerular filtration rate and is not undergoing dialysis. Which result would give the nurse the most concern? a. Albumin level of 2.5 g/dL b. Phosphorus level of 5 mg/dL c. Sodium level of 135 mmol/L d. Potassium level of 5.5 mmol/L Protein restriction is necessary with chronic renal failure due to the buildup of waste products from protein breakdown. The nurse would be concerned with the low albumin level since this indicates that the protein in the diet is not enough for the client's metabolic needs. The electrolyte values are not related to the protein-restricted diet.

A

A child diagnosed with severe autistic spectrum disorder has the nursing diagnosis disturbed personal identity. Which outcome would best address this client diagnosis? A. The client will name own body parts as separate from others by day five. B. The client will establish a means of communicating personal needs by discharge. C. The client will initiate social interactions with caregivers by day four. D. The client will not harm self or others by discharge. Rationale: An appropriate outcome for this client is to name own body parts as separate from others. The nurse should assist the client in the recognition of separateness during self-care activities, such as dressing and feeding. The long-term goal for disturbed personal identity is for the client to develop an ego identity.

A

A client admitted with hepatopulmonary syndrome is experiencing dyspnea but does not want oxygen increased because the client's nose keeps bleeding from it. The client becomes agitated when discussing this with the nurse. The client's oxygen saturation is 92%. What intervention by the nurse is best? a. Instruct the client to sit in as upright a position as possible. b. Tell the client that humidity can be added, but that the oxygen must be worn. c. Document the client's refusal in the chart, and call the health care provider. d. Call the health care provider to request an extra dose of the client's diuretic. The client with hepatopulmonary syndrome is often dyspneic. Because the oxygen saturation is not significantly low, the nurse should first allow the client to sit upright to see if that helps. If the client remains dyspneic, or if the oxygen saturation drops further, the nurse should investigate adding humidity to the oxygen and seeing whether the client will tolerate that. The other two options may be beneficial, but they are not the best choices. If the client is comfortable, his or her agitation will decrease; this will improve respiratory status.

A

A client had a paracentesis 1 hour ago. Which assessment finding requires action by the nurse? a.Urine output of 20 mL/hr b.Systolic blood pressure increase of 10 mm Hg c.Respiratory rate drop from 18 to 14 d.A 3-pound drop in weight Rapid removal of ascitic fluid causes decreased abdominal pressure, which can contribute to hypovolemia. This can be manifested by a decrease in urine output to below 30 mL/hr. The nurse would expect the client's weight to drop as fluid is removed. A decrease in respiratory rate indicates that breathing has been made easier by the procedure. A slight increase in systolic blood pressure is insignificant.

A

A client has a serum potassium level of 6.5 mmol/L, a serum creatinine level of 2 mg/dL, and a urine output of 350 mL/day. What is the best action by the nurse? a. Place the client on a cardiac monitor immediately. b. Teach the client to limit high-potassium foods. c. Continue to monitor the client's intake and output. d. Ask to have the laboratory redraw the blood specimen. The priority action by the nurse should be to check the cardiac status with a monitor. High potassium levels can lead to dysrhythmias. The other choices are logical nursing interventions for acute kidney injury but not the best immediate action.

A

A client has been missing some scheduled hemodialysis sessions. Which intervention is most important for the nurse to implement? a. Discussing with the client his or her acceptance of the disease b. Discussing with the client the option of peritoneal dialysis c. Rescheduling the sessions to another day or another time d. Stressing to the client the importance of going to the sessions Some people on dialysis retreat into complete or partial denial of the disease and the need for treatment. They may deny the need for dialysis and/or may not adhere to drug therapy and diet restrictions. Providing support as the client struggles to accept the disease is an important step in ensuring compliance with the dialysis regimen. The nurse should explore scheduling options, but missing so many sessions cues the nurse that a bigger problem than just scheduling is involved. The nurse should provide education, but simply stressing the need for dialysis will not help the client accept it. Peritoneal dialysis, with its technical demands on the client and partner, probably is not an option for a client who appears noncompliant with hemodialysis.

A

A client is admitted to the hospital with a serum creatinine level of 2 mg/dL. When taking the client's history, which question does the nurse ask first? a."Do you take any nonprescription medications?" b."Does anyone in your family have kidney disease?" c."Do you have yearly blood work done?" d."Is your diet low in protein?" Acute renal failure can be caused by certain medications considered to have a nephrotoxic effect, such as NSAIDs and acetaminophen. Asking the client whether he or she takes any nonprescription drugs can help determine which medication(s) might have contributed to the problem. A family history is important but is not as vital as assessing for nephrotoxic agents that the client may have ingested. Yearly blood work might reveal a trend in kidney function, but again would not be as important. A diet low in protein would not be an important factor to assess.

A

A client is admitted with jaundice and suspected hepatitis B. Which intervention does the nurse add to the client's care plan? a. Encourage rest during this period. b. Assist the client with ambulation. c. Place the client on a clear liquid diet. d. Administer PRN prochlorperazine maleate (Compazine). During the icteric phase, the client is encouraged to rest. Rest reduces the metabolic demands of the liver and promotes hepatic cell regeneration. The client may or may not need assistance with ambulation. The diet should be high in carbohydrates and calories for energy; clear liquids may be needed if the client

A

A client is bleeding from esophageal varices. The health care provider is arranging sclerotherapy for the client. Before the client goes to interventional radiology, the nurse prepares to administer which medication? a.Terlipressin (Glypressin) b.Enoxaparin (Lovenox) c.Lactulose (Heptalac) d.Spironolactone (Aldactone) Terlipressin is a vasoactive drug that works by reducing portal pressure, which decreases bleeding. These drugs are often given in conjunction with sclerotherapies. Enoxaparin is a low-molecular-weight heparin, which would be contraindicated in a client with bleeding problems. Lactulose helps rid the body of ammonia. Aldactone is a diuretic.

A

A client is diagnosed with hepatitis B. Which information does the nurse include in the teaching plan as a priority? a."Avoid drinking any alcohol until the doctor says you can." b."You will need aggressive control of your serum lipids." c."Once your lab work returns to normal, you can donate blood again." d."Wash your hands well after handling meat and shellfish." Alcohol has a hepatotoxic effect, and clients with any liver disease should not drink it. Serum lipids need control in clients with fatty liver. Once a client has hepatitis B, he or she should not donate blood or organs. Handling contaminated shellfish is a cause of hepatitis A infection.

A

A client is hospitalized with acute pancreatitis. The nursing assistant reports to the nurse that when a blood pressure cuff was applied, the client's hand had a spasm. Which additional finding does the nurse correlate with this condition? a. Serum calcium, 5.8 mg/dL b. Serum sodium, 166 mEq/L c. Serum creatinine, 0.9 mg/dL d. Serum potassium, 4.2 mEq/dL Spasm of the hand when a blood pressure cuff is applied (Trousseau's sign) is indicative of hypocalcemia. The client's calcium level is low. The sodium level is high, but that is not related to Trousseau's sign. Creatinine and potassium levels are normal.

A

A client is receiving lactulose (Heptalac). Which laboratory value leads the nurse to intervene? a. Serum potassium, 2.6 mEq/L b. Serum sodium, 132 mEq/L c.Serum glucose, 108 mg/dL d. Blood urea nitrogen, 16 mg/dL Lactulose can cause the client to have several loose stools daily. The nurse should monitor for hypokalemia and dehydration. This client's potassium level is low, indicating hypokalemia. The serum sodium level is slightly low, but hyponatremia is not a complication of lactulose therapy. The blood glucose is slightly high, but this is unrelated. Blood urea nitrogen (BUN) is normal.

A

A client is refusing to take lactulose (Heptalac) because of diarrhea. Which is the nurse's best response to this client? a."Diarrhea is expected; that's how your body gets rid of ammonia." b."You may take Kaopectate liquid daily for loose stools." c."Do not take any more of the medication until your stools firm up." d."We will need to send a stool specimen to the laboratory." The purpose of administering lactulose to this client is to help ammonia leave the circulatory system through the colon. Lactulose draws water into the bowel with its high osmotic gradient, thereby producing a laxative effect and subsequently evacuating ammonia from the bowel. The client must understand that this is an expected and therapeutic effect for him or her to remain compliant. The other statements are inaccurate.

A

A client is taking furosemide (Lasix) 40 mg/day for management of chronic kidney disease (CKD). To detect the positive effect of the medication, what action of the nurse is best? a. Obtain daily weights of the client. b. Auscultate heart and breath sounds. c. Palpate the client's abdomen. d. Assess the client's diet history. Furosemide (Lasix) is a loop diuretic that helps reduce fluid overload and hypertension in clients with early stages of CKD. One kilogram of weight equals about 1 liter of fluid retained in the client, so daily weights are necessary to monitor the response of the client to the medication. Heart and breath sounds should be assessed if there is fluid retention, as in heart failure. Palpation of the client's abdomen is not necessary, but the nurse should check for edema. The diet history of the client would be helpful to assess electrolyte replacement since potassium is lost with this diuretic, but this does not assess the effect of the medication.

A

A client requests information on several medications in order to make an informed choice about management of depression. A nurse should provide this information to facilitate which ethical principle? A. Autonomy B. Beneficence C. Nonmaleficence D. Justice Rationale: The nurse should provide the information to support the client's autonomy. A client who is capable of making independent choices should be permitted to do so. In instances when clients are incapable of making informed decisions, a legal guardian or representative would be asked to give consent.

A

A client who had been hospitalized with pancreatitis is being discharged with home health services. The client is severely weakened after this illness. Which nursing intervention is the highest priority in conserving the client's strength? A Limiting the client's activities to one floor of the home Correct B Instructing the client to take an as-needed (PRN) sleeping medication at night C Arranging for the client to have a nutritional consult to assess the client's diet D Asking the health care provider for a request for PRN nasal oxygen Limiting the client's activities to one floor of the home will prevent tiring the client with stair climbing. Taking a PRN sleeping medication may not necessarily increase the client's strength level or conserve strength; also, the client may not be experiencing difficulty sleeping. Arranging for a nutritional consult or placing the client on PRN nasal oxygen will not necessarily result in an increase in the client's strength level or conserve strength; no information suggests that the client has any history of breathing difficulties.

A

A client with acute kidney injury had normal assessments 1 hour ago. Now the nurse finds that the client's respiration rate is 44 breaths/min and the client is restless. Which assessment does the nurse perform? a.Obtain an oxygen saturation level. b.Send blood for a creatinine level. c.Assess the client for dehydration. d.Perform a bedside blood glucose. A complication of acute kidney injury is pulmonary edema. Manifestations of this include tachypnea; frothy, blood-tinged sputum; and tachycardia, anxiety, and crackles. The nurse needs to obtain an oxygen saturation, listen to the client's lungs, and notify the health care provider, so that treatment can be started. The other interventions are not helpful.

A

A client with chronic kidney disease (CKD) is experiencing nausea, vomiting, visual changes, and anorexia. Which action by the nurse is best? a. Check the client's digoxin (Lanoxin) level. b. Administer an anti-nausea medication. c. Ask if the client is able to eat crackers. d. Get a referral to a gastrointestinal provider. These signs and symptoms are indications of digoxin (Lanoxin) toxicity. The nurse should check the level of this medication. Administering antiemetics, asking if the client can eat, and obtaining a referral to a specialist all address the client's symptoms but do not lead to the cause of the symptoms.

A

A client with hepatitis C is being treated with ribavirin (Copegus). What nursing action takes priority? a.Educating the client on ways to remain complaint with the drug regimen b.Teaching the client that transient muscle aching is a common side effect c. Ensuring that the client returns to the clinic each week for follow-up care d. Showing the client how to take and record a radial pulse for 1 minute Treatment with ribavirin takes up to 48 weeks, making compliance a serious issue. The nurse should work with the client on a strategy to remain compliant for this length of time. The other actions are not warranted.

A

A male client with chronic kidney disease (CKD) is refusing to take his medication and has missed two hemodialysis appointments. What is the best initial action for the nurse? a. Discuss what the treatment regimen means to him. b. Refer the client to a mental health nurse practitioner. c. Reschedule the appointments to another date and time. d. Discuss the option of peritoneal dialysis. The initial action for the nurse is to assess anxiety, coping styles, and the client's acceptance of the required treatment for CKD. The client may be in denial of the diagnosis. While rescheduling hemodialysis appointments may help, and referral to a mental health practitioner and the possibility of peritoneal dialysis are all viable options, assessment of the client's acceptance of the treatment should come first.

A

A nurse assesses a client who is recovering from a paracentesis 1 hour ago. Which assessment finding requires action by the nurse? a. Urine output via indwelling urinary catheter is 20 mL/hr b. Blood pressure increases from 110/58 to 120/62 mm Hg c. Respiratory rate decreases from 18 to 14 breaths/min d. A decrease in the client's weight by 6 kg Rapid removal of ascetic fluid causes decreased abdominal pressure, which can contribute to hypovolemia. This can be manifested by a decrease in urine output to below 30 mL/hr. A slight increase in systolic blood pressure is insignificant. A decrease in respiratory rate indicates that breathing has been made easier by the procedure. The nurse would expect the client's weight to drop as fluid is removed. Six kilograms is less than 3 pounds and is expected.

A

A nurse assesses a client who is recovering from an open Whipple procedure. Which action should the nurse perform first? a. Assess the client's endotracheal tube with 40% FiO2. b. Insert an indwelling Foley catheter to gravity drainage. c. Place the client's nasogastric tube to low intermittent suction. d. Start lactated Ringer's solution through an intravenous catheter. Using the ABCs, airway and oxygenation status should always be assessed first, so checking the endotracheal tube is the first action. Next, the nurse should start the IV line (circulation). After that, the Foley catheter can be inserted and the nasogastric tube can be set.

A

A nurse assesses an adolescent client diagnosed with conduct disorder who, at the age of 8, was sentenced to juvenile detention. How should the nurse interpret this assessment data? A. Childhood-onset conduct disorder is more severe than the adolescent-onset type, and these individuals likely develop antisocial personality disorder in adulthood. B. Childhood-onset conduct disorder is caused by a difficult temperament, and the child is likely to outgrow these behaviors by adulthood. C. Childhood-onset conduct disorder is diagnosed only when behaviors emerge before the age of 5, and, therefore, improvement is likely. D. Childhood-onset conduct disorder has no treatment or cure, and children diagnosed with this disorder are likely to develop progressive oppositional defiant disorder. Rationale: The nurse should determine that childhood-onset conduct disorder is more severe than adolescent-onset type. These individuals are likely to develop antisocial personality disorder in adulthood. Individuals with this subtype are usually boys and frequently display physical aggression and have disturbed peer relationships.

A

A nurse assesses clients at a community health fair. Which client is at greatest risk for the development of hepatitis B? a. A 20-year-old college student who has had several sexual partners b. A 46-year-old woman who takes acetaminophen daily for headaches c. A 63-year-old businessman who travels frequently across the country d. An 82-year-old woman who recently ate raw shellfish for dinner Hepatitis B can be spread through sexual contact, needle sharing, needle sticks, blood transfusions, hemodialysis, acupuncture, and the maternal-fetal route. A person with multiple sexual partners has more opportunities to contract the infection. Hepatitis B is not transmitted through medications, casual contact with other travelers, or raw shellfish. Although an overdose of acetaminophen can cause liver cirrhosis, this is not associated with hepatitis B. Hepatitis E is found most frequently in international travelers. Hepatitis A is spread through ingestion of contaminated shellfish.

A

After an adolescent diagnosed with attention deficit-hyperactivity disorder (ADHD) begins methylphenidate (Ritalin) therapy, a nurse notes that the adolescent loses 10 pounds in a 2-month period. What is the best explanation for this weight loss? A. The pharmacological action of Ritalin causes a decrease in appetite. B. Hyperactivity seen in ADHD causes increased caloric expenditure. C. Side effects of Ritalin cause nausea, and, therefore, caloric intake is decreased. D. Increased ability to concentrate allows the client to focus on activities rather than food Rationale: The pharmacological action of Ritalin causes a decrease in appetite, which often leads to weight loss. Methylphenidate is a central nervous symptom stimulant that serves to increase attention span, control hyperactive behaviors, and improve learning ability.

A

A nurse cares for a client who is prescribed lactulose (Heptalac). The client states, "I do not want to take this medication because it causes diarrhea." How should the nurse respond? a. "Diarrhea is expected; that's how your body gets rid of ammonia." b. "You may take Kaopectate liquid daily for loose stools." c. "Do not take any more of the medication until your stools firm up." d. "We will need to send a stool specimen to the laboratory." The purpose of administering lactulose to this client is to help ammonia leave the circulatory system through the colon. Lactulose draws water into the bowel with its high osmotic gradient, thereby producing a laxative effect and subsequently evacuating ammonia from the bowel. The client must understand that this is an expected and therapeutic effect for him or her to remain compliant. The nurse should not suggest administering anything that would decrease the excretion of ammonia or holding the medication. There is no need to send a stool specimen to the laboratory because diarrhea is the therapeutic response to this medication.

A

A nurse cares for a client with diabetes mellitus who is prescribed metformin (Glucophage) and is scheduled for an intravenous urography. Which action should the nurse take first? a. Contact the provider and recommend discontinuing the metformin. b. Keep the client NPO for at least 6 hours prior to the examination. c. Check the client's capillary artery blood glucose and administer prescribed insulin. d. Administer intravenous fluids to dilute and increase the excretion of dye. Metformin can cause lactic acidosis and renal impairment as the result of an interaction with the dye. This drug must be discontinued for 48 hours before the procedure and not started again after the procedure until urine output is well established. The client's health care provider needs to provide alternative therapy for the client until the metformin can be resumed. Keeping the client NPO, checking the client's blood glucose, and administering intravenous fluids should be part of the client's plan of care, but are not the priority, as the examination should not occur while the client is still taking metformin.

A

A nurse cares for a client with end-stage pancreatic cancer. The client asks, "Why is this happening to me?" How should the nurse respond? a. "I don't know. I wish I had an answer for you, but I don't." b. "It's important to keep a positive attitude for your family right now." c. "Scientists have not determined why cancer develops in certain people." d. "I think that this is a trial so you can become a better person because of it." The client is not asking the nurse to actually explain why the cancer has occurred. The client may be expressing his or her feelings of confusion, frustration, distress, and grief related to this diagnosis. Reminding the client to keep a positive attitude for his or her family does not address the client's emotions or current concerns. The nurse should validate that there is no easy or straightforward answer as to why the client has cancer. Telling a client that cancer is a trial is untrue and may diminish the client-nurse relationship.

A

A nurse cares for a client with hepatopulmonary syndrome who is experiencing dyspnea with oxygen saturations at 92%. The client states, "I do not want to wear the oxygen because it causes my nose to bleed. Get out of my room and leave me alone!" Which action should the nurse take? a. Instruct the client to sit in as upright a position as possible. b. Add humidity to the oxygen and encourage the client to wear it. c. Document the client's refusal, and call the health care provider. d. Contact the provider to request an extra dose of the client's diuretic. The client with hepatopulmonary syndrome is often dyspneic. Because the oxygen saturation is not significantly low, the nurse should first allow the client to sit upright to see if that helps. If the client remains dyspneic, or if the oxygen saturation drops further, the nurse should investigate adding humidity to the oxygen and seeing whether the client will tolerate that. The other two options may be beneficial, but they are not the best choices. If the client is comfortable, his or her agitation will decrease; this will improve respiratory status.

A

A nurse prepares to discharge a client with chronic pancreatitis. Which question should the nurse ask to ensure safety upon discharge? a. "Do you have a one- or two-story home?" b. "Can you check your own pulse rate?" c. "Do you have any alcohol in your home?" d. "Can you prepare your own meals?" A client recovering from chronic pancreatitis should be limited to one floor until strength and activity increase. The client will need a bathroom on the same floor for frequent defecation. Assessing pulse rate and preparation of meals is not specific to chronic pancreatitis. Although the client should be encouraged to stop drinking alcoholic beverages, asking about alcohol availability is not adequate to assess this client's safety.

A

A nurse teaches a client with hepatitis C who is prescribed ribavirin (Copegus). Which statement should the nurse include in this client's discharge education? a. "Use a pill organizer to ensure you take this medication as prescribed." b. "Transient muscle aching is a common side effect of this medication." c. "Follow up with your provider in 1 week to test your blood for toxicity." d. "Take your radial pulse for 1 minute prior to taking this medication." Treatment of hepatitis C with ribavirin takes up to 48 weeks, making compliance a serious issue. The nurse should work with the client on a strategy to remain compliant for this length of time. Muscle aching is not a common side effect. The client will be on this medication for many weeks and does not need a blood toxicity examination. There is no need for the client to assess his or her radial pulse prior to taking the medication.

A

A psychiatric nurse working on an inpatient unit receives a call asking if an individual has been a client in the facility. Which nursing response reflects appropriate legal and ethical obligations? A. The nurse refuses to give any information to the caller, citing rules of confidentiality. B. The nurse hangs up on the caller. C. The nurse confirms that the person has been at the facility but adds no additional information. D. The nurse suggests that the caller speak to the client's therapist. Rationale: The most appropriate action by the nurse is to refuse to give any information to the caller. Admission to the facility would be considered protected health information (PHI) and should not be disclosed by the nurse without prior client consent.

A

The nurse is assessing health fair participants for risks for hepatitis. The nurse recognizes which client as being at greatest risk for developing hepatitis B? a.College student who has had several sexual partners b.Woman who takes acetaminophen daily for headaches c.Businessman who travels frequently d.Older woman who has eaten raw shellfish Hepatitis B can be spread through sexual contact, needle sharing, needle sticks, blood transfusions, hemodialysis, acupuncture, and the maternal-fetal route. A person with multiple sexual partners has more opportunities to contract the infection.

A

The nurse is meeting a client post-liver transplantation for the first time and notices a tremor as they shake hands. The client states this has not happened before. Which action by the nurse is most appropriate? a.Conduct a thorough assessment, then notify the surgeon of the findings. b. Review today's laboratory work, including liver function studies. c. Assess the client's vital signs, and offer acetaminophen if the client is febrile. d. Perform an assessment of the client's gross and fine motor skills. The client may be exhibiting asterixis. Any sign of deteriorating neurologic function could indicate that the new liver is not working properly. The surgeon must be notified, but first the nurse should conduct a thorough assessment of the client. Reviewing today's laboratory work is important, but this is not the best option because the client's liver could have deteriorated after the laboratory work was drawn. Clients with any type of liver problem should not take acetaminophen. Assessing the client's fine and gross motor skills is part of an assessment.

A

The nurse is teaching a client with chronic kidney disease (CKD) about the sodium restriction needed in the diet to prevent edema and hypertension. Which statement by the client indicates more teaching is needed? a. "I am thrilled that I can continue to eat fast food." b. "I will cut out bacon with my eggs every morning." c. "My cooking style will change by not adding salt." d. "I will probably lose weight by cutting out potato chips." Fast food restaurants usually serve food that is high in sodium. This statement indicates that more teaching needs to occur. The other statements show a correct understanding of the teaching.

A

When planning care for a client, which medication classification should a nurse recognize as effective in the treatment of Tourette's syndrome? A. Neuroleptic medications B. Anti-manic medications C. Tricyclic antidepressant medications D. Monoamine oxidase inhibitor medications Rationale: The nurse should recognize that neuroleptic (antipsychotic) medications are effective in the treatment of Tourette's syndrome. These medications are used to reduce the severity of tics and are most effective when combined with psychosocial therapy.

A

Without authorization, a nurse administers an extra dose of narcotic tranquilizer to an agitated client. The nurse's coworker observes this action but does nothing for fear of retaliation. What is the ethical interpretation of the coworker's lack of involvement? A. Taking no action is still considered an unethical action by the coworker. B. Taking no action releases the coworker from ethical responsibility. C. Taking no action is advised when potential adverse consequences are foreseen. D. Taking no action is acceptable, because the coworker is only a bystander. Rationale: The coworker's lack of involvement can be interpreted as an unethical action. The coworker is experiencing an ethical dilemma in which a decision needs to be made between two unfavorable alternatives. The coworker has a responsibility to report any observed unethical actions.

A

What is an Ascites?

A collection of free fluid within the peritoneal cavity caused by increased hydrostatic pressure from portal hypertension. The result is a fluid shift from the vascular system into the abdomen, a form of "third spacing." Thus, the patient may have hypovolemia and edema at the same time.

Bleeding esophageal varices are a what?

A life-threatening medical emergency. Severe blood loss may occur, resulting in shock from hypovolemia. The bleeding may be either hematemesis (vomiting blood) or melena (black, tarry stools). Loss of consciousness may occur before any observed bleeding. Any activity that increases abdominal pressure may increase the likelihood of a variceal bleed including lifting or vigorous physical exercise. In addition, chest trauma or dry, hard food in the esophagus can cause bleeding.

What is Autism Spectrum Disorder (ASD)?

A neurological disorder that interferes with normal brain development, affects the way sensory input is processed and sensed. Results in deficits in communication, learning, reasoning, and social functioning. Results in serious learning, communication, and interpersonal problems.

What is Portal Hypertension?

A persistent increase in pressure within the portal vein greater than 5 mm Hg, is a major complication of cirrhosis. It results from increased resistance to or obstruction (blockage) of the flow of blood through the portal vein and its branches. The blood meets resistance to flow and seeks collateral (alternative) venous channels around the high-pressure area

What are the characteristics of Oppositional Defiant Disorder (ODD)?

A persistent pattern of angry mood and defiant behavior that occurs more frequently than is usually observed in comparable age and development. Interferes w/ social, education, vocational activities passive/aggressive behaviors: Stubbornness, procrastination, disobedience, negativism, carelessness, testing limits, resistance to direction, unwilling to cooperate, running away, school avoidance, under achievement, temper tantrums, fighting, argumentative, impaired inter personal relationship (often friendless)

The nurse is caring for five clients on the medical-surgical unit. Which clients would the nurse consider to be at risk for post-renal acute kidney injury (AKI)? (Select all that apply.) a. Man with prostate cancer b. Woman with blood clots in the urinary tract c. Client with ureterolithiasis d. Firefighter with severe burns e. Young woman with lupus Urine flow obstruction, such as prostate cancer, blood clots in the urinary tract, and kidney stones (ureterolithiasis), causes post-renal AKI. Severe burns would be a pre-renal cause. Lupus would be an intrarenal cause for AKI.

A, B, C

Which of the following risk factors, if noted during a family history assessment, should a nurse associate with the development of IDD? (Select all that apply.) A. A family history of Tay-Sachs disease B. Childhood meningococcal infection C. Deprivation of nurturance and social contact D. History of maternal multiple motor and verbal tics E. A diagnosis of maternal major depressive disorder Rationale: The nurse should recognize a family history of Tay-Sachs disease, childhood meningococcal infections, and deprivation of nurturance and social contact as risk factors that would predispose a child to IDD. There are five major predisposing factors of IDD: hereditary factors, early alterations in embryonic development, pregnancy and perinatal factors, medical conditions acquired in infancy or childhood, and environmental influences and other mental disorders.

A, B, C

A client is undergoing hemodialysis. The client's blood pressure at the beginning of the procedure was 136/88 mm Hg, and now it is 110/54 mm Hg. What actions should the nurse perform to maintain blood pressure? (Select all that apply.) a. Adjust the rate of extracorporeal blood flow. b. Place the client in the Trendelenburg position. c. Stop the hemodialysis treatment. d. Administer a 250-mL bolus of normal saline. e. Contact the health care provider for orders. Hypotension occurs often during hemodialysis treatments as a result of vasodilation from the warmed dialysate. Modest decreases in blood pressure, as is the case with this client, can be maintained with rate adjustment, Trendelenburg positioning, and a fluid bolus. If the blood pressure drops considerably after two boluses and cooling dialysate, the hemodialysis can be stopped and the health care provider contacted.

A, B, D

A client is unsure of the decision to undergo peritoneal dialysis (PD) and wishes to discuss the advantages of this treatment with the nurse. Which statements by the nurse are accurate regarding PD? (Select all that apply.) a. "You will not need vascular access to perform PD." b. "There is less restriction of protein and fluids." c. "You will have no risk for infection with PD." d. "You have flexible scheduling for the exchanges." e. "It takes less time than hemodialysis treatments." PD is based on exchanges of waste, fluid, and electrolytes in the peritoneal cavity. There is no need for vascular access. Protein is lost in the exchange, which allows for more protein and fluid in the diet. There is flexibility in the time for exchanges, but the treatment takes a longer period of time compared to hemodialysis. There still is risk for infection with PD, especially peritonitis.

A, B, D

A nurse reviews a client's laboratory results. Which results from the client's urinalysis should the nurse identify as normal? (Select all that apply.) a. pH: 6 b. Specific gravity: 1.015 c. Protein: 1.2 mg/dL d. Glucose: negative e. Nitrate: small f. Leukocyte esterase: positive The pH, specific gravity, and glucose are all within normal ranges. The other values are abnormal.

A, B, D

A nurse teaches a client with polycystic kidney disease (PKD). Which statements should the nurse include in this client's discharge teaching? (Select all that apply.) a. "Take your blood pressure every morning." b. "Weigh yourself at the same time each day." c. "Adjust your diet to prevent diarrhea." d. "Contact your provider if you have visual disturbances." e. "Assess your urine for renal stones." A client who has PKD should measure and record his or her blood pressure and weight daily, limit salt intake, and adjust dietary selections to prevent constipation. The client should notify the provider if urine smells foul or has blood in it, as these are signs of a urinary tract infection or glomerular injury. The client should also notify the provider if visual disturbances are experienced, as this is a sign of a possible berry aneurysm, which is a complication of PKD. Diarrhea and renal stones are not manifestations or complications of PKD; therefore, teaching related to these concepts would be inappropriate.

A, B, D

A nurse assesses a male client who has symptoms of cirrhosis. Which questions should the nurse ask to identify potential factors contributing to this laboratory result? (Select all that apply.) a. "How frequently do you drink alcohol?" b. "Have you ever had sex with a man?" c. "Do you have a family history of cancer?" d. "Have you ever worked as a plumber?" e. "Were you previously incarcerated?" When assessing a client with suspected cirrhosis, the nurse should ask about alcohol consumption, including amount and frequency; sexual history and orientation (specifically men having sex with men); illicit drug use; history of tattoos; and history of military service, incarceration, or work as a firefighter, police officer, or health care provider. A family history of cancer and work as a plumber do not put the client at risk for cirrhosis.

A, B, E

A nurse delegates hygiene care for a client who has advanced cirrhosis to an unlicensed nursing personnel (UAP). Which statements should the nurse include when delegating this task to the UAP? (Select all that apply.) a. "Apply lotion to the client's dry skin areas." b. "Use a basin with warm water to bathe the client." c. "For the client's oral care, use a soft toothbrush." d. "Provide clippers so the client can trim the fingernails." e. "Bathe with antibacterial and water-based soaps." Clients with advanced cirrhosis often have pruritus. Lotion will help decrease itchiness from dry skin. A soft toothbrush should be used to prevent gum bleeding, and the client's nails should be trimmed short to prevent the client from scratching himself or herself. These clients should use cool, not warm, water on their skin, and should not use excessive amounts of soap.

A, C, D

A nurse plans care for a client who has hepatopulmonary syndrome. Which interventions should the nurse include in this client's plan of care? (Select all that apply.) a. Oxygen therapy b. Prone position c. Feet elevated on pillows d. Daily weights e. Physical therapy Care for a client who has hepatopulmonary syndrome should include oxygen therapy, the head of bed elevated at least 30 degrees or as high as the client wants to improve breathing, elevated feet to decrease dependent edema, and daily weights. There is no need to place the client in a prone position, on the client's stomach. Although physical therapy may be helpful to a client who has been hospitalized for several days, physical therapy is not an intervention specifically for hepatopulmonary syndrome.

A, C, D

An infection control nurse develops a plan to decrease the number of health care professionals who contract viral hepatitis at work. Which ideas should the nurse include in this plan? (Select all that apply.) a. Policies related to consistent use of Standard Precautions b. Hepatitis vaccination mandate for workers in high-risk areas c. Implementation of a needleless system for intravenous therapy d. Number of sharps used in client care reduced where possible e. Postexposure prophylaxis provided in a timely manner Nurses should always use Standard Precautions for client care, and policies should reflect this. Needleless systems and reduction of sharps can help prevent hepatitis. Postexposure prophylaxis should be provided immediately. All health care workers should receive the hepatitis vaccinations that are available.

A, C, D, E

The infection control nurse wants to decrease the number of health care professionals who contract viral hepatitis at work. Which actions does the nurse initiate? (Select all that apply.) a. Strengthen policies related to consistent use of Standard Precautions. b. Mandate hepatitis vaccination for workers in high-risk areas. c. Implement a needleless system for IV therapy. d. Reduce the number of "sharps" needed for client care where possible. e. Provide post exposure prophylaxis in a timely manner. Nurses should always use Standard Precautions for client care, and policies should reflect this. Needleless systems and reduction of sharps can help prevent hepatitis. Postexposure prophylaxis should be provided immediately. All health care workers should receive the hepatitis vaccinations that are available.

A, C, D, E

A client is hospitalized in the oliguric phase of acute kidney injury (AKI) and is receiving tube feedings. The nurse is teaching the client's spouse about the kidney-specific formulation for the enteral solution compared to standard formulas. What components should be discussed in the teaching plan? (Select all that apply.) a. Lower sodium b. Higher calcium c. Lower potassium d. Higher phosphorus e. Higher calories Many clients with AKI are too ill to meet caloric goals and require tube feedings with kidney-specific formulas that are lower in sodium, potassium, and phosphorus, and higher in calories than are standard formulas.

A, C, E

A nurse is caring for a postoperative 70-kg client who had major blood loss during surgery. Which findings by the nurse should prompt immediate action to prevent acute kidney injury? (Select all that apply.) a. Urine output of 100 mL in 4 hours b. Urine output of 500 mL in 12 hours c. Large amount of sediment in the urine d. Amber, odorless urine e. Blood pressure of 90/60 mm Hg The low urine output, sediment, and blood pressure should be reported to the provider. Postoperatively, the nurse should measure intake and output, check the characteristics of the urine, and report sediment, hematuria, and urine output of less than 0.5 mL/kg/hour for 3 to 4 hours. A urine output of 100 mL is low, but a urine output of 500 mL in 12 hours should be within normal limits. Perfusion to the kidneys is compromised with low blood pressure. The amber odorless urine is normal.

A, C, E

After disturbing the peace, an aggressive, disoriented, unkempt, homeless individual is escorted to an emergency department. The client threatens suicide. Which of the following criteria would enable a physician to consider involuntary commitment? (Select all that apply.) A. Being dangerous to others B. Being homeless C. Being disruptive to the community D. Being gravely disabled and unable to meet basic needs E. Being suicidal Rationale: The physician could consider involuntary commitment when a client is dangerous to others, gravely disabled, or is suicidal. If the physician determines that the client is mentally incompetent, consent should be obtained from the legal guardian or court-approved guardian or conservator. A hospital administrator may give permission for involuntary commitment when time does not permit court intervention.

A, D, E

A client has undergone the Whipple procedure (radical pancreaticoduodenectomy) for pancreatic cancer. Which precautionary measures does the nurse implement to prevent potential complications? (Select all that apply.) A. Check blood glucose often. Correct B. Check bowel sounds and stools. Correct C. Ensure that drainage color is clear. D. Monitor mental status. Correct E. Place the client in the supine position. Glucose should be checked often to monitor for diabetes mellitus. Bowels sounds and stools should be checked to monitor for bowel obstruction. A change in mental status or level of consciousness could be indicative of hemorrhage. Clear, colorless, bile-tinged drainage or frank blood with increased output may indicate disruption or leakage of a site of anastomosis. The client should be placed in semi-Fowler's position to reduce tension on the suture line and the anastomosis site and to optimize lung expansion.

A,B,D

A client previously diagnosed with liver cirrhosis visits the medical clinic. What assessment findings does the nurse expect in this client? Select all that apply. A. Ecchymosis B. Soft abdomen C. Moist, clammy skin D. Jaundice E. Ankle edema F. Fever

A,D,E

Which statement by a client with alcohol-induced cirrhosis indicates the need for further teaching? a."I cannot drink any alcohol at all anymore." b. "I need to avoid protein in my diet." c."I should not take over-the-counter medications." d."I should eat small, frequent, balanced meals." Based on the degree of liver involvement and decreased function, protein intake may have to be decreased. However, some protein is necessary for the synthesis of albumin and normal healing. The other statements indicate accurate understanding of self-care measures for this client.

B

A client has an IQ of 47. Which nursing diagnosis best addresses a client problem associated with this degree of IDD? A. Risk for injury R/T self-mutilation B. Altered social interaction R/T non-adherence to social convention C. Altered verbal communication R/T delusional thinking D. Social isolation R/T severely decreased gross motor skills Rationale: The appropriate nursing diagnosis associated with this degree of IDD is altered social interaction R/T non-adherence to social convention. A client with an IQ of 47 would be diagnosed with moderate intellectual developmental disorder and may also experience some limitations in speech communications.

B

A client has been diagnosed with hepatitis A. The nurse evaluates that teaching regarding the disease is understood when the client makes which statement? a."Some medications have been known to induce hepatitis A." b."I may have been exposed when we ate shrimp last weekend." c."I may have been infected through a recent blood transfusion." d."My infection with Epstein-Barr virus can co-infect me with hepatitis A." The route of acquisition of hepatitis A infection is through close personal contact or ingestion of contaminated water or shellfish. The other statements are not accurate.

B

A client has liver cancer. Which statement by the client about treatment options demonstrates an accurate understanding? a."I guess it's a good thing that surgery is usually so successful." b."I choose hepatic arterial infusion of chemo to limit side effects." c."Because I have only local metastases, I am thinking about transplant." d. "This disease is so rare, no wonder no good treatments are available." Hepatic arterial chemotherapy infusion allows chemotherapeutic agents to be delivered directly into the liver tumor, limiting systemic side effects. Surgery is not usually successful because the cancer is frequently widespread when detected. Transplant is considered only for primary liver tumors that have not metastasized. The lack of successful treatments is due not to rarity, but rather to the fact that generally the cancer has already spread when found.

B

A client is 12 hours post-kidney transplantation. The nurse notes that the client has put out 2000 mL of urine in 10 hours. Which assessment does the nurse carry out first? a.Skin turgor b.Blood pressure c.Serum blood urea nitrogen (BUN) level d.Weight of the client After transplantation, the client may have diuresis. Excessive diuresis might cause hypotension. Hypotension needs to be prevented because it can reduce blood flow and oxygen to the new kidney, threatening graft survival. The other assessments can give information about fluid balance, but hypotension is the main concern here, so the nurse needs to check the client's blood pressure, then notify the provider.

B

A client is admitted with acute kidney injury (AKI) and a urine output of 2000 mL/day. What is the major concern of the nurse regarding this client's care? a. Edema and pain b. Electrolyte and fluid imbalance c. Cardiac and respiratory status d. Mental health status This client may have an inflammatory cause of AKI with proteins entering the glomerulus and holding the fluid in the filtrate, causing polyuria. Electrolyte loss and fluid balance is essential. Edema and pain are not usually a problem with fluid loss. There could be changes in the client's cardiac, respiratory, and mental health status if the electrolyte imbalance is not treated.

B

A client is admitted with end-stage cirrhosis and severe vomiting. Which problem should the nurse monitor the client most carefully for? a. Intrahepatic bile stasis b. Bleeding esophageal varices c. Decreased excretion of bilirubin d. Accumulation of ascites in the abdomen The portal hypertension that accompanies end-stage cirrhosis predisposes the client to esophageal varices. These varices can rupture from increased pressure in the esophagus caused by coughing or vomiting. Bleeding varices can be life threatening. None of the other assessments take priority over monitoring for bleeding from esophageal varices.

B

A client is having a peritoneal dialysis treatment. The nurse notes an opaque color to the effluent. What is the priority action by the nurse? a. Warm the dialysate solution in a microwave before instillation. b. Take a sample of the effluent and send to the laboratory. c. Flush the tubing with normal saline to maintain patency of the catheter. d. Check the peritoneal catheter for kinking and curling. An opaque or cloudy effluent is the first sign of peritonitis. A sample of the effluent would need to be sent to the laboratory for culture and sensitivity in order to administer the correct antibiotic. Warming the dialysate in a microwave and flushing the tubing are not safe actions by the nurse. Checking the catheter for obstruction is a viable option but will not treat the peritonitis.

B

A client is in the emergency department after a motor vehicle crash, and the nurse notices a "steering wheel mark" across the client's chest. Which action by the nurse is most appropriate? a. Ask the client where in the car he or she was during the crash. b. Assess the client by gently palpating the abdomen for tenderness. c. Notify the laboratory to come draw blood for blood type and crossmatch. d. Place the client on the stretcher in reverse Trendelenburg position. The liver is often injured by a steering wheel in a motor vehicle crash. Because the client's chest was marked by the steering wheel, the nurse should perform an abdominal assessment. Assessing the client's position in the crash is not needed because of the steering wheel imprint. The client may or may not need a blood transfusion. The client does not need to be in reverse Trendelenburg position.

B

A client is scheduled for a paracentesis. Which activity does the nurse delegate to the unlicensed assistive personnel? a. Have the client sign the informed consent form. b. Assist the client to void before the procedure. c. Help the client lie flat in bed, on the right side. d. Get the client into a chair after the procedure. For safety, the client should void just before a paracentesis. The nurse or the provider should have the client sign the consent form. The proper position for a paracentesis is sitting upright in bed or, alternatively, sitting on the side of the bed and leaning over the bedside table. The client will be on bedrest after the procedure.

B

A client just had a paracentesis. Which nursing intervention is a priority for this client? a.Monitor urine output. b.Maintain bedrest as per protocol. c.Position the client flat in bed. d.Secure the trocar to the abdomen with tape. After a paracentesis, the client should remain on bedrest with the head of the bed elevated. A client with liver dysfunction is at risk for bleeding, and bedrest decreases this risk. Clients with liver dysfunction must have intake and output monitored, but this is not the priority after this procedure. A drain may be placed for short-term therapy in some clients.

B

A client who had a liver transplant a month ago is admitted with fever and tachycardia. Which medication does the nurse prepare to administer to this client? a.Ceftriaxone (Rocephin) b.Cyclosporine (Sandimmune) c.Azithromycin (Zithromax) d.Ribavirin (Copegus) This client is showing signs of transplant rejection, which is treated with immune suppressive drugs, such as cyclosporine. Ceftriaxone and azithromycin are antibiotics. Ribavirin is used to treat hepatitis C.

B

A client who underwent liver transplantation 2 weeks ago reports a temperature of 101° F (38.3° C) and right flank pain. Which is the nurse's best response? a."The anti-rejection drugs you are taking made you susceptible to infection." b."You should go to the hospital immediately to have your new liver checked out." c."You should take an additional dose of cyclosporine today." d."Take acetaminophen (Tylenol) every 4 hours until you feel better." Fever, right quadrant or flank pain, and jaundice are signs of liver transplant rejection; the client should be admitted to the hospital as soon as possible for intervention. The other statements are not appropriate.

B

A client with chronic hypertension is seen in the clinic. Which assessment indicates that the client's hypertension is not under control? a.Heart rate of 55 beats/min b.Serum creatinine level of 1.9 mg/dL c.Blood glucose level of 128 mg/dL d.Irregular heart sounds Increased blood pressure damages the delicate capillaries in the glomerulus and eventually results in acute kidney injury. An elevated serum creatinine level is a manifestation of this. Heart rate, blood glucose level, and irregular heart sounds are not correlated with acute kidney injury.

B

A client with polycystic kidney disease and hypertension is prescribed a diuretic for blood pressure control. Which statement by the client indicates the need for further teaching regarding these orders? a."I will weigh myself every day at the same time." b."I will drink only 1 liter of fluid each day." c."I will avoid aspirin and aspirin-containing drugs." d."I will avoid nonsteroidal anti-inflammatory drugs." Diuretics for blood pressure control can lead to fluid volume depletion and can decrease blood flow to the kidney, further decreasing renal function. The client should be instructed to drink at least 2500 mL/24 hr. NSAIDs should be used cautiously because they can reduce kidney blood flow. Aspirin products increase the risk for bleeding and should be avoided.

B

A nurse assesses a client with polycystic kidney disease (PKD). Which assessment finding should alert the nurse to immediately contact the health care provider? a. Flank pain b. Periorbital edema c. Bloody and cloudy urine d. Enlarged abdomen Periorbital edema would not be a finding related to PKD and should be investigated further. Flank pain and a distended or enlarged abdomen occur in PKD because the kidneys enlarge and displace other organs. Urine can be bloody or cloudy as a result of cyst rupture or infection.

B

A nurse cares for a client who has cirrhosis of the liver. Which action should the nurse take to decrease the presence of ascites? a. Monitor intake and output. b. Provide a low-sodium diet. c. Increase oral fluid intake. d. Weigh the client daily. A low-sodium diet is one means of controlling abdominal fluid collection. Monitoring intake and output does not control fluid accumulation, nor does weighing the client. These interventions merely assess or monitor the situation. Increasing fluid intake would not be helpful.

B

A nurse cares for a client who is recovering from an open Whipple procedure. Which action should the nurse take? a. Clamp the nasogastric tube. b. Place the client in semi-Fowler's position. c. Assess vital signs once every shift. d. Provide oral rehydration. Postoperative care for a client recovering from an open Whipple procedure should include placing the client in a semi-Fowler's position to reduce tension on the suture line and anastomosis sites, setting the nasogastric tube to low suction to remove free air buildup and pressure, assessing vital signs frequently to assess fluid and electrolyte complications, and providing intravenous fluids.

B

A nurse cares for a client who is scheduled for a paracentesis. Which intervention should the nurse delegate to an unlicensed assistive personnel (UAP)? a. Have the client sign the informed consent form. b. Assist the client to void before the procedure. c. Help the client lie flat in bed on the right side. d. Get the client into a chair after the procedure. For safety, the client should void just before a paracentesis. The nurse or the provider should have the client sign the consent form. The proper position for a paracentesis is sitting upright in bed or, alternatively, sitting on the side of the bed and leaning over the bedside table. The client will be on bedrest after the procedure.

B

A nurse cares for a client with a urine specific gravity of 1.018. Which action should the nurse take? a. Evaluate the client's intake and output for the past 24 hours. b. Document the finding in the chart and continue to monitor. c. Obtain a specimen for a urine culture and sensitivity. d. Encourage the client to drink more fluids, especially water. This specific gravity is within the normal range for urine. There is no need to evaluate the client's intake and output, obtain a urine specimen, or increase fluid intake.

B

After teaching a client with hypertension secondary to renal disease, the nurse assesses the client's understanding. Which statement made by the client indicates a need for additional teaching? a. "I can prevent more damage to my kidneys by managing my blood pressure." b. "If I have increased urination at night, I need to drink less fluid during the day." c. "I need to see the registered dietitian to discuss limiting my protein intake." d. "It is important that I take my antihypertensive medications as directed." The client should not restrict fluids during the day due to increased urination at night. Clients with renal disease may be prescribed fluid restrictions. These clients should be assessed thoroughly for potential dehydration. Increased nocturnal voiding can be decreased by consuming fluids earlier in the day. Blood pressure control is needed to slow the progression of renal dysfunction. When dietary protein is restricted, refer the client to the registered dietitian as needed.

B

A nurse cares for a client with hepatic portal-systemic encephalopathy (PSE). The client is thin and cachectic in appearance, and the family expresses distress that the client is receiving little dietary protein. How should the nurse respond? a. "A low-protein diet will help the liver rest and will restore liver function." b. "Less protein in the diet will help prevent confusion associated with liver failure." c. "Increasing dietary protein will help the client gain weight and muscle mass." d. "Low dietary protein is needed to prevent fluid from leaking into the abdomen." A low-protein diet is ordered when serum ammonia levels increase and/or the client shows signs of PSE. A low-protein diet helps reduce excessive breakdown of protein into ammonia by intestinal bacteria. Encephalopathy is caused by excess ammonia. A low-protein diet has no impact on restoring liver function. Increasing the client's dietary protein will cause complications of liver failure and should not be suggested. Increased intravascular protein will help prevent ascites, but clients with liver failure are not able to effectively synthesize dietary protein.

B

A nurse cares for a client with hepatitis C. The client's brother states, "I do not want to contract this infection, so I will not go into his hospital room." How should the nurse respond? a. "If you wear a gown and gloves, you will not get this virus." b. "Viral hepatitis is not spread through casual contact." c. "This virus is only transmitted through a fecal specimen." d. "I can give you an update on your brother's status from here." Although family members may be afraid that they will contract hepatitis C, the nurse should educate the client's family about how the virus is spread. Viral hepatitis, or hepatitis C, is spread via blood-to-blood transmission and is associated with illicit IV drug needle sharing, blood and organ transplantation, accidental needle sticks, unsanitary tattoo equipment, and sharing of intranasal cocaine paraphernalia. Wearing a gown and gloves will not decrease the transmission of this virus. Hepatitis C is not spread through casual contact or a fecal specimen. The nurse would be violating privacy laws by sharing the client's status with the brother.

B

A nurse provides health screening for a community health center with a large population of African-American clients. Which priority assessment should the nurse include when working with this population? a. Measure height and weight. b. Assess blood pressure. c. Observe for any signs of abuse. d. Ask about medications. All interventions are important for the visiting nurse to accomplish. However, African Americans have a high rate of hypertension leading to end-stage renal disease. Each encounter that the nurse has with an African-American client provides a chance to detect hypertension and treat it. If the client is already on antihypertensive medication, assessing blood pressure monitors therapy.

B

A nurse reviews a female client's laboratory results. Which results from the client's urinalysis should the nurse recognize as abnormal? a. pH 5.6 b. Ketone bodies present c. Specific gravity of 1.020 d. Clear and yellow color Ketone bodies are by-products of incomplete metabolism of fatty acids. Normally no ketones are present in urine. Ketone bodies are produced when fat sources are used instead of glucose to provide cellular energy. A pH between 4.6 and 8, specific gravity between 1.005 and 1.030, and clear yellow urine are normal findings for a female client's urinalysis.

B

A nurse reviews the health history of a client with an oversecretion of renin. Which disorder should the nurse correlate with this assessment finding? a. Alzheimer's disease b. Hypertension c. Diabetes mellitus d. Viral hepatitis Renin is secreted when special cells in the distal convoluted tubule, called the macula densa, sense changes in blood volume and pressure. When the macula densa cells sense that blood volume, blood pressure, or blood sodium levels are low, renin is secreted. Renin then converts angiotensinogen into angiotensin I. This leads to a series of reactions that cause secretion of the hormone aldosterone. This hormone increases kidney reabsorption of sodium and water, increasing blood pressure, blood volume, and blood sodium levels. Inappropriate or excessive renin secretion is a major cause of persistent hypertension. Renin has no impact on Alzheimer's disease, diabetes mellitus, or viral hepatitis.

B

A thin, cachectic-appearing client has hepatic portal-systemic encephalopathy (PSE). The family expresses distress that the client is receiving so little protein in the diet. Which explanation by the nurse is most appropriate? a."A low-protein diet will help the liver rest and will restore liver function." b."Less protein in the diet will help with the confusion." c."Despite looking so thin, protein will not help with weight gain." d."Less protein is needed to prevent fluid from leaking into the abdomen." A low-protein diet is ordered when serum ammonia levels increase and/or the client shows signs of PSE. A low-protein diet helps reduce excessive breakdown of protein into ammonia by intestinal bacteria. Encephalopathy is caused by excess ammonia. The other statements are not correct.

B

After teaching a client who has been diagnosed with hepatitis A, the nurse assesses the client's understanding. Which statement by the client indicates a correct understanding of the teaching? a. "Some medications have been known to cause hepatitis A." b. "I may have been exposed when we ate shrimp last weekend." c. "I was infected with hepatitis A through a recent blood transfusion." d. "My infection with Epstein-Barr virus can co-infect me with hepatitis A." The route of acquisition of hepatitis A infection is through close personal contact or ingestion of contaminated water or shellfish. Hepatitis A is not transmitted through medications, blood transfusions, or Epstein-Barr virus. Toxic and drug-induced hepatitis is caused from exposure to hepatotoxins, but this is not a form of hepatitis A. Hepatitis B can be spread through blood transfusions. Epstein-Barr virus causes a secondary infection that is not associated with hepatitis A.

B

After teaching a client with early polycystic kidney disease (PKD) about nutritional therapy, the nurse assesses the client's understanding. Which statement made by the client indicates a correct understanding of the teaching? a. "I will take a laxative every night before going to bed." b. "I must increase my intake of dietary fiber and fluids." c. "I shall only use salt when I am cooking my own food." d. "I'll eat white bread to minimize gastrointestinal gas." Clients with PKD often have constipation, which can be managed with increased fiber, exercise, and drinking plenty of water. Laxatives should be used cautiously. Clients with PKD should be on a restricted salt diet, which includes not cooking with salt. White bread has a low fiber count and would not be included in a high-fiber diet.

B

After teaching a client with renal cancer who is prescribed temsirolimus (Torisel), the nurse assesses the client's understanding. Which statement made by the client indicates a correct understanding of the teaching? a. "I will take this medication with food and plenty of water." b. "I shall keep my appointment at the infusion center each week." c. "I'll limit my intake of green leafy vegetables while on this medication." d. "I must not take this medication if I have an infection or am feeling ill." Temsirolimus is administered as a weekly intravenous infusion. This medication blocks protein that is needed for cell division and therefore inhibits cell cycle progression. This medication is not taken orally, and clients do not need to follow a specific diet.

B

An emergency department nurse assesses a client with kidney trauma and notes that the client's abdomen is tender and distended and blood is visible at the urinary meatus. Which prescription should the nurse consult the provider about before implementation? a. Assessing vital signs every 15 minutes b. Inserting an indwelling urinary catheter c. Administering intravenous fluids at 125 mL/hr d. Typing and crossmatching for blood products Clients with blood at the urinary meatus should not have a urinary catheter inserted via the urethra before additional diagnostic studies are done. The urethra could be torn. The nurse should question the provider about the need for a catheter; if one is needed, the provider can insert a suprapubic catheter. The nurse should monitor the client's vital signs closely, send blood for type and crossmatch in case the client needs blood products, and administer intravenous fluids.

B

An emergency room nurse assesses a client after a motor vehicle crash. The nurse notices a "steering wheel mark" across the client's chest. Which action should the nurse take? a. Ask the client where in the car he or she was sitting during the crash. b. Assess the client by gently palpating the abdomen for tenderness. c. Notify the laboratory to draw blood for blood type and crossmatch. d. Place the client on the stretcher in reverse Trendelenburg position. The liver is often injured by a steering wheel in a motor vehicle crash. Because the client's chest was marked by the steering wheel, the nurse should perform an abdominal assessment. Assessing the client's position in the crash is not needed because of the steering wheel imprint. The client may or may not need a blood transfusion. The client does not need to be in reverse Trendelenburg position.

B

Cognitive Level: Application A child has been diagnosed with autistic spectrum disorder. The distraught mother cries out, "I'm such a terrible mother. What did I do to cause this?" Which nursing response is most appropriate? A "Researchers really don't know what causes autistic spectrum disorder, but the relationship between autistic disorder and fetal alcohol syndrome is being explored." B "Poor parenting doesn't cause autistic spectrum disorder. Research has shown that abnormalities in brain structure or function are to blame. This is beyond your control." C "Research has shown that the mother appears to play a greater role in the development of autistic spectrum disorder than the father." D "Lack of early infant bonding with the mother has shown to be a cause of autistic spectrum disorder. Did you breastfeed or bottle-feed?" Rationale: The most appropriate response by the nurse is to explain to the parent that autistic spectrum disorder is believed to be caused by abnormalities in brain structure or function, not poor parenting. Autism occurs in approximately 11.3 per 1,000 children and is about 4.5 times more likely to occur in boys than girls.

B

Group therapy is strongly encouraged, but not mandatory, in an inpatient psychiatric unit. The unit manager's policy is that clients can make a choice about whether or not to attend group therapy. Which ethical principle does the unit manager's policy preserve? A. Justice B. Autonomy C. Veracity D. Beneficence Rationale: The unit manager's policy regarding voluntary client participation in group therapy preserves the ethical principle of autonomy. The principle of autonomy presumes that individuals are capable of making independent decisions for themselves and that health-care workers must respect these decisions.

B

On assessment of a client with polycystic kidney disease (PKD), which finding is of greatest concern to the nurse? a.Flank pain b.Periorbital edema c.Bloody and cloudy urine d.Enlarged abdomen Flank pain and a distended or enlarged abdomen occur in PKD because the kidneys enlarge and displace other organs. Urine can be bloody or cloudy owing to cyst rupture or infection. Periorbital edema would not be a finding related to PKD and should be investigated further.

B

The nurse is caring for a client with acute pancreatitis. Which nursing intervention best reduces discomfort for the client? a. Administering morphine sulfate IV every 4 to 6 hours as needed b. Maintaining NPO status for the client with IV fluids c. Providing small, frequent feedings, with no concentrated sweets d. Placing the client in semi-Fowler's position at elevation of 30 degrees The client should be kept NPO to reduce GI activity and reduce pancreatic enzyme production. IV fluids should be used to prevent dehydration. The client may need a nasogastric (NG) tube. Pain medications should be given around the clock and more frequently than every 4 to 6 hours. A fetal position with legs drawn up to the chest will promote comfort.

B

The nurse is caring for a client with chronic pancreatitis. Which instruction by the nurse is most appropriate? a. "You will need to limit your protein intake." b. "We need to call the dietitian to get help in planning your diet." c. "You cannot eat concentrated sweets any longer." d. "Try to eat less red meat and more chicken and fish." A client with chronic pancreatitis needs 4000 to 6000 calories per day for optimum nutrition and healing. The client may have additional restrictions if he or she has other health problems such as diabetes. The nurse should collaborate with the registered dietitian to help the client plan nutritional intake.

B

The nurse is providing dietary teaching to a client who was just started on peritoneal dialysis (PD). Which instruction does the nurse provide to this client regarding protein intake? a."Your protein needs will not change, but you may take more fluids." b."You will need more protein now because some protein is lost by dialysis." c."Your protein intake will be adjusted according to your predialysis weight." d."You no longer need to be on protein restriction." When renal disease has progressed and requires treatment with dialysis, increased protein is required in the diet to compensate for protein losses through peritoneal dialysis. The other statements are inaccurate.

B

The nurse is reviewing a client's history. Which statement by the client indicates a need for health teaching? a."I drink 1 to 2 glasses of red wine a week." b."Because of my arthritis, I take a lot of Tylenol." c."One of my cousins died of liver cancer 10 years ago." d."I got a hepatitis vaccine before traveling last year." Acetaminophen (Tylenol) can cause liver damage if taken in large amounts. Clients should be taught not to exceed 4000 mg/day of acetaminophen. The nurse should teach the client about this limitation and should explore other drug options with the client to manage his or her arthritis pain. The other statements do not necessarily require health teaching by the nurse.

B

The nurse is teaching the main principles of hemodialysis to a client with chronic kidney disease. Which statement by the client indicates a need for further teaching by the nurse? a. "My sodium level changes by movement from the blood into the dialysate." b. "Dialysis works by movement of wastes from lower to higher concentration." c. "Extra fluid can be pulled from the blood by osmosis." d. "The dialysate is similar to blood but without any toxins." Dialysis works using the passive transfer of toxins by diffusion. Diffusion is the movement of molecules from an area of higher concentration to an area of lower concentration. The other statements show a correct understanding about hemodialysis.

B

The nurse recognizes that fetor hepaticus is consistent with which assessment finding? a.Purpuric lesions on the extremities b.A fruity or musty breath odor c.Warm and bright red palms d.Jaundice of the sclera Fetor hepaticus is a distinctive breath odor that presents with chronic liver disease. The client's breath has a fruity or musty odor. The other statements do not apply to fetor hepaticus.

B

The nursing assistant is helping a client who has advanced cirrhosis with a bath and other hygiene. Which action by the assistant requires intervention by the registered nurse? a. Helping the client apply lotion to dry skin areas b. Giving the client a basin of warm water and soap to use c. Providing a soft toothbrush for oral care d. Helping the client keep nails trimmed short Clients with advanced cirrhosis often have pruritus. These clients should use cool, not warm, water on their skin, and should not use excessive amounts of soap. The other actions are appropriate.

B

Which behavioral approach should a nurse use when caring for children diagnosed with disruptive behavior disorders? A. Involving parents in designing and implementing the treatment process B. Reinforcing positive actions to encourage repetition of desirable behaviors C. Providing opportunities to learn appropriate peer interactions D. Administering psychotropic medications to improve quality of life Rationale: The nurse should reinforce positive actions to encourage repetition of desirable behaviors when caring for children diagnosed with disruptive behavior disorder. Behavior therapy is based on the concepts of classical conditioning and operant conditioning.

B

Which finding should a nurse expect when assessing a child diagnosed with separation anxiety disorder? A. The child has a history of antisocial behaviors. B. The child's mother is diagnosed with an anxiety disorder. C. The child previously had an extroverted temperament. D. The child's mother and father have an inconsistent parenting style. Rationale: The nurse should expect to find a mother diagnosed with an anxiety disorder when assessing a child with separation anxiety. Some parents instill anxiety in their children by being overprotective or by exaggerating dangers. Research studies speculate that there is a hereditary influence in the development of separation anxiety disorder.

B

Which is an example of an intentional tort? A. A nurse fails to assess a client's obvious symptoms of neuroleptic malignant syndrome. B. A nurse physically places an irritating client in four-point restraints. C. A nurse makes a medication error and does not report the incident. D. A nurse gives patient information to an unauthorized person. Rationale: A tort, which can be intentional or unintentional, is a violation of civil law in which an individual has been wronged. A nurse who intentionally physically places an irritating client in restraints has touched the client without consent and has committed an intentional tort.

B

Which laboratory findings does the nurse recognize as potentially causing complications of liver disease? a.Elevated aspartate transaminase (AST) and lactate dehydrogenase (LDH) levels b.Elevated prothrombin time and international normalized ratio (INR) c.Decreased serum albumin and serum globulin levels d.Decreased serum alkaline phosphatase and alanine aminotransferase (ALT) levels Elevated prothrombin time and INR are indications of clotting disturbances and alert the nurse to the increased possibility of hemorrhage. The other values do not necessarily place the client at increased risk for complications

B

Which nursing intervention related to self-care would be most appropriate for a teenager diagnosed with moderate IDD? A. Meeting all of the client's self-care needs to avoid injury to the client B. Providing simple directions and praising client's independent self-care efforts C. Avoid interfering with the client's self-care efforts in order to promote autonomy D. Encouraging family to meet the client's self-care needs to promote bonding Rationale: Providing simple directions and praise is an appropriate intervention for a teenager diagnosed with moderate IDD. Individuals with moderate mental retardation can perform some activities independently and may be capable of academic skill to a second-grade level.

B

Which potential client should a nurse identify as a candidate for involuntarily commitment? A. The client living under a bridge in a cardboard box B. The client threatening to commit suicide C. The client who never bathes and wears a wool hat in the summer D. The client who eats waste out of a garbage can Rationale: The nurse should identify the client threatening to commit suicide as eligible for involuntary commitment. The suicidal client who refuses treatments is in danger and needs emergency treatment.

B

Which situation exemplifies both assault and battery? A. The nurse becomes angry, calls the client offensive names, and withholds treatment. B. The nurse threatens to "tie down" the client and then does so, against the client's wishes. C. The nurse hides the client's clothes and medicates the client to prevent elopement. D. The nurse restrains the client without just cause and communicates this to family. Rationale: The nurse in this situation has committed both the acts of assault and battery. Assault refers to an action that results in fear and apprehension that the person will be touched without consent. Battery is the touching of another person without consent.

B

A nurse collaborates with an unlicensed assistive personnel (UAP) to provide care for a client who is in the healing phase of acute pancreatitis. Which statements focused on nutritional requirements should the nurse include when delegating care for this client? (Select all that apply.) a. "Do not allow the client to eat between meals." b. "Make sure the client receives a protein shake." c. "Do not allow caffeine-containing beverages." d. "Make sure the foods are bland with little spice." e. "Do not allow high-carbohydrate food items." During the healing phase of pancreatitis, the client should be provided small, frequent, moderate- to high-carbohydrate, high-protein, low-fat meals. Protein shakes can be provided to supplement the diet. Foods and beverages should not contain caffeine and should be bland.

B, C, D

A nurse assesses a client who is recovering from a Whipple procedure. Which clinical manifestations alert the nurse to a complication from this procedure? (Select all that apply.) a. Clay-colored stools b. Substernal chest pain c. Shortness of breath d. Lack of bowel sounds or flatus e. Urine output of 20 mL/6 hr Myocardial infarction (chest pain), pulmonary embolism (shortness of breath), adynamic ileus (lack of bowel sounds or flatus), and renal failure (urine output of 20 mL/6 hr) are just some of the complications for which the nurse must assess the client after the Whipple procedure. Clay-colored stools are associated with cholecystitis and are not a complication of a Whipple procedure.

B, C, D, E

A nurse is giving discharge instructions to a client recently diagnosed with chronic kidney disease (CKD). Which statements made by the client indicate a correct understanding of the teaching? (Select all that apply.) a. "I can continue to take antacids to relieve heartburn." b. "I need to ask for an antibiotic when scheduling a dental appointment." c. "I'll need to check my blood sugar often to prevent hypoglycemia." d. "The dose of my pain medication may have to be adjusted." e. "I should watch for bleeding when taking my anticoagulants." In discharge teaching, the nurse must emphasize that the client needs to have an antibiotic prophylactically before dental procedures to prevent infection. There may be a need for dose reduction in medications if the kidney is not excreting them properly (antacids with magnesium, antibiotics, antidiabetic drugs, insulin, opioids, and anticoagulants).

B, C, D, E

An emergency room nurse assesses a client with potential liver trauma. Which clinical manifestations should alert the nurse to internal bleeding and hypovolemic shock? (Select all that apply.) a. Hypertension b. Tachycardia c. Flushed skin d. Confusion e. Shallow respirations Symptoms of hemorrhage and hypovolemic shock include hypotension, tachycardia, tachypnea, pallor, diaphoresis, cool and clammy skin, and confusion.

B, D

Which of the following findings should a nurse identify that would contribute to a client's development of ADHD? (Select all that apply.) A. The client's father was a smoker. B. The client was born 7 weeks premature. C. The client is lactose intolerant. D. The client has a sibling diagnosed with ADHD. E. The client has been diagnosed with dyslexia. Rationale: The nurse should identify that premature birth and having a sibling diagnosed with ADHD would predispose a client to the development of ADHD. Research indicates evidence of genetic influences in the etiology of ADHD. Studies also indicate that environmental influences, such as lead exposure and diet, can be linked with the development of ADHD.

B, D

A nurse assesses a client who is recovering from a nephrostomy. Which assessment findings should alert the nurse to urgently contact the health care provider? (Select all that apply.) a. Clear drainage b. Bloody drainage at site c. Client reports headache d. Foul-smelling drainage e. Urine draining from site After a nephrostomy, the nurse should assess the client for complications and urgently notify the provider if drainage decreases or stops, drainage is cloudy or foul-smelling, the nephrostomy sites leaks blood or urine, or the client has back pain. Clear drainage is normal. A headache would be an unrelated finding.

B, D, E

The nurse is teaching a client with diabetes mellitus how to prevent or delay chronic kidney disease (CKD). Which client statements indicate a lack of understanding of the teaching? (Select all that apply.) a. "I need to decrease sodium, cholesterol, and protein in my diet." b. "My weight should be maintained at a body mass index of 30." c. "Smoking should be stopped as soon as I possibly can." d. "I can continue to take an aspirin every 4 to 8 hours for my pain." e. "I really only need to drink a couple of glasses of water each day." Weight should be maintained at a body mass index (BMI) of 22 to 25. A BMI of 30 indicates obesity. The use of nonsteroidal anti-inflammatory drugs such as aspirin should be limited to the lowest time at the lowest dose due to interference with kidney blood flow. The client should drink at least 2 liters of water daily. Diet adjustments should be made by restricting sodium, cholesterol, and protein. Smoking causes constriction of blood vessels and decreases kidney perfusion, so the client should stop smoking.

B, D, E

A nurse teaches a client who is recovering from acute pancreatitis. Which statements should the nurse include in this client's teaching? (Select all that apply.) a. "Take a 20-minute walk at least 5 days each week." b. "Attend local Alcoholics Anonymous (AA) meetings weekly." c. "Choose whole grains rather than foods with simple sugars." d. "Use cooking spray when you cook rather than margarine or butter." e. "Stay away from milk and dairy products that contain lactose." f. "We can talk to your doctor about a prescription for nicotine patches." The client should be advised to stay sober, and AA is a great resource. The client requires a low-fat diet, and cooking spray is low in fat compared with butter or margarine. If the client smokes, he or she must stop because nicotine can precipitate an exacerbation. A nicotine patch may help the client quit smoking. The client must rest until his or her strength returns. The client requires high carbohydrates and calories for healing; complex carbohydrates are not preferred over simple ones. Dairy products do not cause a problem.

B, D, F

The nurse is assessing a client with chronic kidney disease (CKD) and pericarditis. What manifestations of pericarditis does the nurse expect to find in the client? Select all that apply. A Mild chest pain B Low-grade fever C Decreased pulse rate D Pericardial friction rub D Halitosis The presence of low-grade fever is a manifestation of pericarditis, the inflammation of the pericardial sac by uremic toxins or infection. A pericardial friction rub is heard on auscultation over the left sternal border, indicating pericarditis or an inflammation of the pericardial sac. The client experiences severe chest pain and an increased pulse rate. Halitosis, or bad breath, is caused by the breakdown of urea into ammonia. The reaction is catalyzed by the enzyme urease, present in the mouth. Halitosis is not a manifestation of pericarditis.

B,D

A client just returned to the nursing unit after having a trans-jugular intrahepatic portal-systemic shunt (TIPS) procedure. Which clinical finding does the nurse expect to observe in this client? a.Decreased level of consciousness b.Decreased urinary volume c.Increased blood pressure d.Increased abdominal girth With TIPS placement, ascitic fluid is routed into the venous system, resulting in vascular volume expansion. An increase in blood pressure is reflective of increased circulating volume. The client should not have the other findings

C

What is the Onset stage?

Begins with precipitating event (such a kidney blockage or decreased cardiac output)

DKA Labs

Blood Glucose >250-600 pH <7.30, HCO3 <15 (metabolic acidosis) serum & urine glucose with large ketones present Potassium (K) High Hyperosmolality NA low CL low Phosphate low Mg low

What is a complication of Portal Hypertension?

Blood flow backs into the spleen, causing splenomegaly (spleen enlargement). Veins in the esophagus, stomach, intestines, abdomen, and rectum become dilated. It can result in ascites (excessive abdominal [peritoneal] fluid), esophageal varices (distended veins), prominent abdominal veins (caput medusae), and hemorrhoids.

What is Turner's & Cullen's sign?

Bruising of the flanks or abdomen near umbilicus, the bruising appears as a blue discoloration, and is a sign of retroperitoneal hemorrhage, or bleeding behind the peritoneum, which is a lining of the abdominal cavity.

A brother calls to speak to his sister, who has been admitted to a psychiatric unit. The nurse connects him to the community phone, and the sister is summoned. Later the nurse realizes that the brother was not on the client's approved call list. What law has the nurse broken? A. The National Alliance for the Mentally Ill Act B. The Tarasoff Ruling C. The Health Insurance Portability and Accountability Act D. The Good Samaritan Law Rationale: The nurse has violated the Health Insurance Portability and Accountability Act (HIPAA) by revealing that the client had been admitted to the psychiatric unit. The nurse should not have provided any information without proper consent from the client.

C

A client has a serum creatinine level of 2.5 mg/dL, a serum potassium level of 6 mmol/L, an arterial pH of 7.32, and a urine output of 250 mL/day. Which phase of acute kidney failure is the client experiencing? a.Intrarenal b.Nonoliguric c.Oliguric d.Postrenal The oliguric phase of acute kidney failure is characterized by the accumulation of nitrogenous wastes, resulting in increasing levels of serum creatinine and potassium, bicarbonate deficit, and decreased or no urine output.Intrarenal and postrenal refer to causes of kidney injury. Nonoliguric is not a classification.

C

A client has been discharged to home after being hospitalized with an acute episode of pancreatitis. The client, who is an alcoholic, is unwilling to participate in Alcoholics Anonymous (AA), and the client's spouse expresses frustration to the home health nurse regarding the client's refusal. What is the nurse's BEST response? A. "Your spouse will sign up for the meetings only when he is ready to deal with his problem." B. "Keep mentioning the AA meetings to your spouse on a regular basis." C. "I'll get you some information on the support group Al-Anon." D. "Tell me more about your frustration with your spouse's refusal to participate in AA." Putting the client's spouse in contact with an Al-Anon support group assists with the spouse's frustration. Telling the spouse that the client will sign up for AA meetings when the client is ready and telling the spouse to keep mentioning AA do not address the spouse's frustration with the client's refusal to participate in AA. Encouraging the spouse to say more about his or her frustration may allow the spouse to vent frustration, but it does not offer any options or solutions.

C

A client is experiencing an attack of acute pancreatitis. Which nursing intervention is the HIGHEST priority for this client? A. Measure intake and output every shift. B. Do not administer food or fluids by mouth. C. Administer opioid analgesic medication. D. Assist the client to assume a position of comfort. For the client with acute pancreatitis, pain relief is the highest priority. Although measuring intake and output, NPO status, and positioning for comfort are all important, they are not the highest priority.

C

A client is placed on fluid restrictions because of chronic kidney disease (CKD). Which assessment finding would alert the nurse that the client's fluid balance is stable at this time? a. Decreased calcium levels b. Increased phosphorus levels c. No adventitious sounds in the lungs d. Increased edema in the legs The absence of adventitious sounds upon auscultation of the lungs indicates a lack of fluid overload and fluid balance in the client's body. Decreased calcium levels and increased phosphorus levels are common findings with CKD. Edema would indicate a fluid imbalance.

C

A client was just admitted to the emergency department for new-onset confusion. As the nurse starts the IV line, the client says he just finished a hemodialysis session. The IV site is bleeding briskly. What action by the nurse takes priority? a.Assess for a bruit and thrill over the vascular access site. b.Draw blood for coagulation studies and white blood cell count. c.Prepare to administer protamine sulfate. d.Hold constant firm pressure with a gauze pad for 5 minutes. To prevent blood clots from forming within the dialyzer or blood tubing, anticoagulation is needed during hemodialysis treatment. The drug used is heparin, which makes the client at risk for hemorrhage for the next 4 to 6 hours. Protamine sulfate is the antidote to heparin, and the nurse should prepare to administer it. Pressure may help, and someone else can apply it while the nurse is getting the medication. Laboratory studies are not needed because the client is at known risk for bleeding from heparin. Assessing the vascular access device does nothing to help the situation.

C

A client who will be receiving electroconvulsive therapy (ECT) must provide informed consent. Which situation should cause a nurse to question the validity of the informed consent? A. The client is paranoid. B. The client is 87 years old. C. The client incorrectly reports his or her spouse's name, date, and time of day. D. The client relies on his or her spouse to interpret the information. Rationale: The nurse should question the validity of informed consent when the client incorrectly reports the spouse's name, date, and time of day. This indicates that this client is disoriented and may not be competent to make informed choices.

C

A mother questions the decreased effectiveness of methylphenidate (Ritalin), prescribed for her child's ADHD. Which nursing response best addresses the mother's concern? A. "The physician will probably switch from Ritalin to a central nervous system stimulant." B. "The physician may prescribe an antihistamine with the Ritalin to improve effectiveness." C. "Your child has probably developed a tolerance to Ritalin and may need a higher dosage." D. "Your child has developed sensitivity to Ritalin and may be exhibiting an allergy." Rationale: The nurse should explain to the mother that the child has probably developed a tolerance to Ritalin and may need a higher dosage. Methylphenidate is a central nervous system stimulant, and tolerance can develop rapidly. Physical and psychological dependence can also occur.

C

A nurse cares for a client who has chronic cirrhosis from substance abuse. The client states, "All of my family hates me." How should the nurse respond? a. "You should make peace with your family." b. "This is not unusual. My family hates me too." c. "I will help you identify a support system." d. "You must attend Alcoholics Anonymous." Clients who have chronic cirrhosis may have alienated relatives over the years because of substance abuse. The nurse should assist the client to identify a friend, neighbor, or person in his or her recovery group for support. The nurse should not minimize the client's concerns by brushing off the client's comment. Attending AA may be appropriate, but this response doesn't address the client's concern. Making peace with the client's family may not be possible. This statement is not client-centered.

C

A nurse cares for a client who has pyelonephritis. The client states, "I am embarrassed to talk about my symptoms." How should the nurse respond? a. "I am a professional. Your symptoms will be kept in confidence." b. "I understand. Elimination is a private topic and shouldn't be discussed." c. "Take your time. It is okay to use words that are familiar to you." d. "You seem anxious. Would you like a nurse of the same gender to care for you?" Clients may be uncomfortable discussing issues related to elimination and the genitourinary area. The nurse should encourage the client to use language that is familiar to the client. The nurse should not make promises that cannot be kept, like keeping the client's symptoms confidential. The nurse must assess the client and cannot take the time to stop the discussion or find another nurse to complete the assessment.

C

A nurse cares for a client who is recovering from a closed percutaneous kidney biopsy. The client states, "My pain has suddenly increased from a 3 to a 10 on a scale of 0 to 10." Which action should the nurse take first? a. Reposition the client on the operative side. b. Administer the prescribed opioid analgesic. c. Assess the pulse rate and blood pressure. d. Examine the color of the client's urine. An increase in the intensity of pain after a percutaneous kidney biopsy is a symptom of internal hemorrhage. A change in vital signs can indicate that hemorrhage is occurring. Before other actions, the nurse must assess the client's hemodynamic status.

C

A nurse cares for a client with acute pancreatitis. The client states, "I am hungry." How should the nurse reply? a. "Is your stomach rumbling or do you have bowel sounds?" b. "I need to check your gag reflex before you can eat." c. "Have you passed any flatus or moved your bowels?" d. "You will not be able to eat until the pain subsides." Paralytic ileus is a common complication of acute pancreatitis. The client should not eat until this has resolved. Bowel sounds and decreased pain are not reliable indicators of peristalsis. Instead, the nurse should assess for passage of flatus or bowel movement.

C

A nurse cares for a middle-aged female client with diabetes mellitus who is being treated for the third episode of acute pyelonephritis in the past year. The client asks, "What can I do to help prevent these infections?" How should the nurse respond? a. "Test your urine daily for the presence of ketone bodies and proteins." b. "Use tampons rather than sanitary napkins during your menstrual period." c. "Drink more water and empty your bladder more frequently during the day." d. "Keep your hemoglobin A1c under 9% by keeping your blood sugar controlled." Clients with long-standing diabetes mellitus are at risk for pyelonephritis for many reasons. Chronically elevated blood glucose levels spill glucose into the urine, changing the pH and providing a favorable climate for bacterial growth. The neuropathy associated with diabetes reduces bladder tone and reduces the client's sensation of bladder fullness. Thus, even with large amounts of urine, the client voids less frequently, allowing stasis and overgrowth of microorganisms. Increasing fluid intake (specifically water) and voiding frequently prevent stasis and bacterial overgrowth. Testing urine and using tampons will not help prevent pyelonephritis. A hemoglobin A1c of 9% is too high.

C

A nurse reviews these laboratory values of a client who returned from kidney transplantation 12 hours ago: Sodium 136 mEq/L Potassium 5 mEq/L Blood urea nitrogen (BUN) 44 mg/dL Serum creatinine 2.5 mg/dL What initial intervention would the nurse anticipate? a. Start hemodialysis immediately. b. Discuss the need for peritoneal dialysis. c. Increase the dose of immunosuppression. d. Return the client to surgery for exploration. The client may need a higher dose of immunosuppressive medication as evidenced by the elevated BUN and serum creatinine levels. This increased dose may reverse the possible acute rejection of the transplanted kidney. The client does not need hemodialysis, peritoneal dialysis, or further surgery at this point.

C

A nurse teaches a client who is recovering from a urography. Which instruction should the nurse include in this client's discharge teaching? a. "Avoid direct contact with your urine for 24 hours until the radioisotope clears." b. "You may have some dribbling of urine for several weeks after this procedure." c. "Be sure to drink at least 3 liters of fluids today to help eliminate the dye faster." d. "Your skin may become slightly yellow from the dye used in this procedure." Dyes used in urography are potentially nephrotoxic. A large fluid intake will help the client eliminate the dye rapidly. Dyes used in urography are not radioactive, the client should not experience any dribbling of urine, and the dye should not change the color of the client's skin.

C

A preschool child diagnosed with autistic spectrum disorder has been engaging in constant head-banging behavior. Which nursing intervention is appropriate? A. Place client in restraints until the aggression subsides. B. Sedate the client with neuroleptic medications. C. Hold client's head steady and apply a helmet. D. Distract the client with a variety of games and puzzles. Rationale: The most appropriate intervention for head banging is to hold the client's head steady and apply a helmet. The helmet is the least restrictive intervention and will serve to protect the client's head from injury.

C

After teaching a client who has plans to travel to a non-industrialized country, the nurse assesses the client's understanding regarding the prevention of viral hepatitis. Which statement made by the client indicates a need for additional teaching? a. "I should drink bottled water during my travels." b. "I will not eat off another's plate or share utensils." c. "I should eat plenty of fresh fruits and vegetables." d. "I will wash my hands frequently and thoroughly." ANS: C The client should be advised to avoid fresh, raw fruits and vegetables because they can be contaminated by tap water. Drinking bottled water, and not sharing plates, glasses, or eating utensils are good ways to prevent illness, as is careful handwashing.

C

Assessment findings reveal that a client with chronic kidney disease is refusing to take prescribed medications because of the "cost." The client also is having difficulty performing activities of daily living and prefers to sleep most of the day. To which health care team member does the nurse refer the client? a.Home health aide b.Physical therapist c.Psychiatric nurse practitioner d.Physician Professionals from many disciplines are resources for the client with renal failure. A psychiatric evaluation may be needed if depressive symptoms are present. Refusing treatment, having difficulty performing activities of daily living, and excessive sleeping could be signs of depression.

C

In planning care for a child diagnosed with autistic spectrum disorder, which would be a realistic client outcome? A. The client will communicate all needs verbally by discharge. B. The client will participate with peers in a team sport by day four. C. The client will establish trust with at least one caregiver by day five. D. The client will perform most self-care tasks independently. Rationale: The most realistic client outcome for a child diagnosed with autistic spectrum disorder is for the client to establish trust with at least one caregiver. Trust should be evidenced by facial responsiveness and eye contact. This outcome relates to the nursing diagnosis impaired social interaction.

C

The charge nurse is orienting a float nurse to an assigned client with an arteriovenous (AV) fistula for hemodialysis in her left arm. Which action by the float nurse would be considered unsafe? a. Palpating the access site for a bruit or thrill b. Using the right arm for a blood pressure reading c. Administering intravenous fluids through the AV fistula d. Checking distal pulses in the left arm The nurse should not use the arm with the AV fistula for intravenous infusion, blood pressure readings, or venipuncture. Compression and infection can result in the loss of the AV fistula. The AV fistula should be monitored by auscultating or palpating the access si

C

The charge nurse of the medical-surgical unit is making staff assignments. Which staff member should be assigned to a client with chronic kidney disease who is exhibiting a low-grade fever and a pericardial friction rub? a. Registered nurse who just floated from the surgical unit b. Registered nurse who just floated from the dialysis unit c. Registered nurse who was assigned the same client yesterday d. Licensed practical nurse with 5 years' experience on this floor The client is exhibiting symptoms of pericarditis, which can occur with chronic kidney disease. Continuity of care is important to assess subtle differences in clients. Therefore, the registered nurse (RN) who was assigned to this client previously should again give care to this client. The float nurses would not be as knowledgeable about the unit and its clients. The licensed practical nurse may not have the education level of the RN to assess for pericarditis.

C

The nurse is assessing a client for asterixis. Which instruction to the client is most appropriate? a."Close your eyes and take turns touching your nose with your fingers." b."Sit on the edge of the bed and hold your legs straight out for 30 seconds." c."Extend your arm, flex your wrist upward, and extend your fingers." d."Say 'EEEEE' while I listen to your lungs in the back on both sides." Asterixis, or liver flap, is a tremor in the client's wrists and fingers. The correct technique for assessing the presence of asterixis is to extend the arm, dorsiflex the wrist, and extend the fingers. The other directions are not related to asterixis.

C

The nurse is assessing a client's alcohol intake to determine whether it is the underlying cause of the client's attacks of pancreatitis. Which question does the nurse ask to elicit this information? A. "Do you usually binge drink?" B. "Do you tend to drink more on holidays or weekends?" C. "Tell me more about your alcohol intake." Correct D. "Estimate how many episodes of binge drinking you do in a week." Asking the client about his or her alcohol intake is the only way that will allow the client to provide information in the client's own words and to the extent that the client wishes to provide it. Asking the client if he or she binge drinks or tends to drink more on holidays or weekends may put the client on the defensive rather than provide the desired information. It has not yet been determined whether the client engages in binge drinking. Awarded

C

The nurse is caring for a client who had undergone a Whipple procedure 2 days previously. The nurse notes that the client's hands and feet are edematous, and urine output has decreased from the previous day. Which intervention does the nurse expect to provide for the client? a. Increase the client's IV fluid infusion rate. b. Monitor the client's blood sugar level every 4 hours. c. Add colloids to the client's IV solutions. d. Reinsert the client's nasogastric (NG) tube. Edema and low urine output following the Whipple procedure most likely are caused by hypoalbuminemia. Low albumin leads to third spacing of fluids and decreased intravascular fluids. As a result, edema and low urine output develop. Adding a colloid solution to the client's IV regimen will help shift edematous fluid from the interstitial space back into the intravascular space. Increasing the client's IV infusion rate will worsen the edema unless additional protein is added. Blood glucose monitoring and NG tubes are not related to this problem.

C

The nurse monitors for which serologic marker in the client who is a carrier of chronic hepatitis B? a. Anti-hepatitis C virus (HCV) antibodies b. Anti-hepatitis B (HBs) antibodies c.Hepatitis B surface antigen (HBsAg) antibodies d.Hepatitis A virus (HAV) antibodies Persistent presence of the serologic marker HBsAg after 6 months indicates a carrier state or chronic hepatitis. The other

C

The nurse recognizes which client as being at greatest risk for the development of carcinoma of the liver? a. Middle-aged client with a history of diabetes mellitus b. Young adult client with a history of blunt liver trauma c. Older adult client with a history of cirrhosis d. Older adult client with malnutrition The risk of contracting a primary carcinoma of the liver is higher in clients with cirrhosis from any cause. The other factors do not increase a person's risk for developing liver cancer.

C

The nurse suspects that a client may have acute pancreatitis as evidenced by which group of laboratory results? A. Deceased calcium, elevated amylase, decreased magnesium B. Elevated bilirubin, elevated alkaline phosphatase C. Elevated lipase, elevated white blood cell count, elevated glucose D. Decreased blood urea nitrogen (BUN), elevated calcium, elevated magnesium Elevated lipase is more specific to a diagnosis of acute pancreatitis. Many pancreatic and non-pancreatic disorders can cause increased serum amylase levels. Bilirubin and alkaline phosphatase levels will be increased only if pancreatitis is accompanied by biliary dysfunction. Usually, calcium and magnesium will be increased and BUN increased, not decreased, in acute pancreatitis.

C

The nursing care plan specifies obtaining abdominal girth measurements each shift. The nurse takes the measurement, but when compared with the previous measurement, the new finding is several millimeters off. Which action by the nurse is best? a. Document the finding in the client's chart. b. Look to see when the client last had a dose of diuretic. c. Ensure that the client's abdomen and flanks are marked with pen. d. Obtain the measurement while the client sits upright. Abdominal girth is measured at the end of exhalation, at the level of the umbilicus, while the client lies flat. To ensure that measurements are taken in the same place each time, the nurse should mark the client's abdomen and flanks with pen. Findings do need to be documented, but this is not the best action when such inconsistency is noted between measurements. Use of a diuretic might decrease ascites, but the best action remains ensuring that measurements are taken in a consistent manner.

C

A nurse cares for a client with autosomal dominant polycystic kidney disease (ADPKD). The client asks, "Will my children develop this disease?" How should the nurse respond? a. "No genetic link is known, so your children are not at increased risk." b. "Your sons will develop this disease because it has a sex-linked gene." c. "Only if both you and your spouse are carriers of this disease." d. "Each of your children has a 50% risk of having ADPKD." Children whose parent has the autosomal dominant form of PKD have a 50% chance of inheriting the gene that causes the disease. ADPKD is transmitted as an autosomal dominant trait and therefore is not gender specific. Both parents do not need to have this disorder.

D

The Immunosuppressant medications given to a patient with a liver transplant at risk for what?

INFECTION! The medications given , are used to prevent and treat organ rejection, significantly increases the patient's risk for infection. Other risk factors include the presence of multiple tubes and intravascular lines, immobility, and prolonged anesthesia.

How do you treat DKA?

Correct fluid and electrolyte disturbances. Provide adequate insulin. Pt and family education and follow up. One of the main goals of treatment of DKA involves correcting high blood glucose levels by injecting insulin as well as replacing fluid lost because of vomiting and urination. HHS does not have ketoacidosis.

A client diagnosed with schizophrenia refuses to take medication, citing the right of autonomy. Under which circumstance would a nurse have the right to medicate the client against the client's wishes? A. A client makes inappropriate sexual innuendos to a staff member. B. A client constantly demands attention from the nurse by begging, "Help me get better." C. A client physically attacks another client after being confronted in group therapy. D. A client refuses to bathe or perform hygienic activities. Rationale: The nurse would have the right to medicate a client against his or her wishes if the client physically attacks another client. This client poses a significant risk to safety and is incapable of making informed choices. The client's refusal to accept treatment can be challenged, because the client is endangering the safety of others.

D

A client has a long history of hypertension. Which category of medications would the nurse expect to be ordered to avoid chronic kidney disease (CKD)? a. Antibiotic b. Histamine blocker c. Bronchodilator d. Angiotensin-converting enzyme (ACE) inhibitor ACE inhibitors stop the conversion of angiotensin I to the vasoconstrictor angiotensin II. This category of medication also blocks bradykinin and prostaglandin, increases renin, and decreases aldosterone, which promotes vasodilation and perfusion to the kidney. Antibiotics fight infection, histamine blockers decrease inflammation, and bronchodilators increase the size of the bronchi; none of these medications helps slow the progression of CKD in clients with hypertension.

D

A client has been placed on enzyme replacement for treatment of chronic pancreatitis. In teaching the client about this therapy, the nurse advises the client not to mix enzyme preparations with foods containing which element? A. Carbohydrates B. High fat C. High fiber D. Protein Enzyme preparations should not be mixed with foods containing protein because the enzymes will dissolve the food into a watery substance. No evidence suggests that enzyme preparations should not be mixed with carbohydrates, food with high fat content, and food with high fiber content.

D

A nurse is caring for a client who is scheduled for a dose of cefazolin and vitamins at this time. Hemodialysis for this client is also scheduled in 60 minutes. Which action by the nurse is best? a. Administer cefazolin since the level of the antibiotic must be maintained. b. Hold the vitamins but administer the cefazolin. c. Hold the cefazolin but administer the vitamins. d. Hold all medications since both cefazolin and vitamins are dialyzable. Both the cefazolin and the vitamins should be held until after the hemodialysis is completed because they would otherwise be removed by the dialysis process.

D

A client hospitalized for worsening kidney injury suddenly becomes restless and agitated. Assessment reveals tachycardia and crackles bilaterally at the bases of the lungs. Which is the nurse's first intervention? a.Begin ultrafiltration. b.Administer an antianxiety agent. c.Place the client on mechanical ventilation. d.Place the client in high Fowler's position. Restlessness, anxiety, tachycardia, dyspnea, and crackles at the bases of the lungs are early manifestations of pulmonary edema, which is a complication of kidney failure. Initial treatment of pulmonary edema consists of placing the client in high Fowler's position and administering oxygen. Mechanical ventilation and ultrafiltration may be indicated if symptoms become worse. An antianxiety agent would not be helpful. Morphine, however, has both vasoactive and sedating effects.

D

A client is assessed by the nurse after a hemodialysis session. The nurse notes bleeding from the client's nose and around the intravenous catheter. What action by the nurse is the priority? a. Hold pressure over the client's nose for 10 minutes. b. Take the client's pulse, blood pressure, and temperature. c. Assess for a bruit or thrill over the arteriovenous fistula. d. Prepare protamine sulfate for administration. Heparin is used with hemodialysis treatments. The bleeding alerts the nurse that too much anticoagulant is in the client's system and protamine sulfate should be administered. Pressure, taking vital signs, and assessing for a bruit or thrill are not as important as medication administration.

D

A client is hemorrhaging from bleeding esophageal varices and has an esophagogastric tube. Which nursing intervention is the priority? a.Keep the client sedated to prevent tube dislodgement. b.Maintain balloon pressure at between 15 and 20 mm Hg. c.Irrigate the gastric lumen with normal saline. d.Maintain the client's airway. Maintaining airway patency is the primary nursing intervention for this client. The nurse suctions oral secretions to prevent aspiration and occlusion of the airway. The client usually is intubated and mechanically ventilated during this procedure. The other interventions are not a priority over airway.

D

A client is receiving an infusion of vasopressin (Pitressin) to treat bleeding esophageal varices. Which client complaint indicates to the nurse that a serious adverse effect of the drug may be occurring? a.Acute nausea and vomiting b. A pounding frontal headache c. Vertigo and syncope d. Midsternal chest pain Midsternal chest pain is indicative of acute angina or myocardial infarction, which can be precipitated by vasopressin. The other side effects do not necessarily indicate that a serious side effect has occurred.

D

A client with severe esophageal varices is scheduled for trans-jugular intrahepatic portal-systemic shunt (TIPS) insertion. The nurse determines that teaching has been effective when the client makes which statement? a."I will be discharged home after I wake up completely." b."The procedure may be painful because I get only light sedation." c."My liver will function normally within 8 hours of placement of the shunt." d."I will be monitored closely for a while after the procedure is over." This procedure is performed in the radiology department under heavy sedation or general anesthesia. The client will be monitored as would any other postoperative client. The other statements are inaccurate.

D

A geriatric client is confused and wandering in and out of every door. Which scenario reflects the least restrictive alternative for this client? A. The client is placed in seclusion. B. The client is placed in a geriatric chair with tray. C. The client is placed in soft Posey restraints. D. The client is monitored by an ankle bracelet. Rationale: The least-restrictive alternative for this client would be monitoring by an ankle bracelet. The client does not pose a direct dangerous threat to self or others, so neither physical restraints nor seclusion would be justified.

D

A nurse assesses a client who is prescribed an infusion of vasopressin (Pitressin) for bleeding esophageal varices. Which clinical manifestation should alert the nurse to a serious adverse effect? a. Nausea and vomiting b. Frontal headache c. Vertigo and syncope d. Mid-sternal chest pain Mid-sternal chest pain is indicative of acute angina or myocardial infarction, which can be precipitated by vasopressin. Nausea and vomiting, headache, and vertigo and syncope are not side effects of vasopressin.

D

A nurse assesses a client who is recovering from a radical nephrectomy for renal cell carcinoma. The nurse notes that the client's blood pressure has decreased from 134/90 to 100/56 mm Hg and urine output is 20 mL for this past hour. Which action should the nurse take? a. Position the client to lay on the surgical incision. b. Measure the specific gravity of the client's urine. c. Administer intravenous pain medications. d. Assess the rate and quality of the client's pulse. The nurse should first fully assess the client for signs of volume depletion and shock, and then notify the provider. The radical nature of the surgery and the proximity of the surgery to the adrenal gland put the client at risk for hemorrhage and adrenal insufficiency. Hypotension is a clinical manifestation associated with both hemorrhage and adrenal insufficiency. Hypotension is particularly dangerous for the remaining kidney, which must receive adequate perfusion to function effectively. Re-positioning the client, measuring specific gravity, and administering pain medication would not provide data necessary to make an appropriate clinical decision, nor are they appropriate interventions at this time.

D

A nurse assesses clients on the medical-surgical unit. Which client should the nurse identify as at high risk for pancreatic cancer? a. A 26-year-old with a body mass index of 21 b. A 33-year-old who frequently eats sushi c. A 48-year-old who often drinks wine d. A 66-year-old who smokes cigarettes Risk factors for pancreatic cancer include obesity, older age, high intake of red meat, and cigarette smoking. Sushi and wine intake are not risk factors for pancreatic cancer.

D

A nurse cares for a client with a urine specific gravity of 1.040. Which action should the nurse take? a. Obtain a urine culture and sensitivity. b. Place the client on restricted fluids. c. Assess the client's creatinine level. d. Increase the client's fluid intake. Normal specific gravity for urine is 1.005 to 1.030. A high specific gravity can occur with dehydration, decreased kidney blood flow (often because of dehydration), and the presence of antidiuretic hormone. Increasing the client's fluid intake would be a beneficial intervention. Assessing the creatinine or obtaining a urine culture would not provide data necessary for the nurse to make a clinical decision.

D

What is Splenomegaly?

It is an enlargement of the spleen.

A nurse reviews the urinalysis results of a client and notes a urine osmolality of 1200 mOsm/L. Which action should the nurse take? a. Contact the provider and recommend a low-sodium diet. b. Prepare to administer an intravenous diuretic. c. Obtain a suction device and implement seizure precautions. d. Encourage the client to drink more fluids. Normal urine osmolality ranges from 300 to 900 mOsm/L. This client's urine is more concentrated, indicating dehydration. The nurse should encourage the client to drink more water. Dehydration can be associated with elevated serum sodium levels. Although a low-sodium diet may be appropriate for this client, this diet change will not have a significant impact on urine osmolality. A diuretic would increase urine output and decrease urine osmolality further. Low serum sodium levels, not elevated serum levels, place the client at risk for seizure activity. These options would further contribute to the client's dehydration or elevate the osmolality.

D

A nurse teaches a client who is recovering from a nephrectomy secondary to kidney trauma. Which statement should the nurse include in this client's teaching? a. "Since you only have one kidney, a salt and fluid restriction is required." b. "Your therapy will include hemodialysis while you recover." c. "Medication will be prescribed to control your high blood pressure." d. "You need to avoid participating in contact sports like football." Clients with one kidney need to avoid contact sports because the kidneys are easily injured. The client will not be required to restrict salt and fluids, end up on dialysis, or have new hypertension because of the nephrectomy.

D

A nursing instructor is teaching about the developmental characteristics of clients diagnosed with moderate intellectual developmental disorder (IDD). Which student statement indicates that further instruction is needed? A. "These clients can work in a sheltered workshop setting." B. "These clients can perform some personal care activities." C. "These clients may have difficulties relating to peers." D. "These clients can successfully complete elementary school." Rationale: The nursing student needs further instruction about moderate IDD, because individuals diagnosed with moderate IDD are capable of academic skill up to a second-grade level. Moderate IDD reflects an IQ range of 35 to 49.

D

An inpatient psychiatric physician refuses to treat clients without insurance and prematurely discharges those whose insurance benefits have expired. Which ethical principle should a nurse determine has been violated based on these actions? A. Autonomy B. Beneficence C. Nonmaleficence D. Justice Rationale: The nurse should determine that the ethical principle of justice has been violated by the physician's actions. The principle of justice requires that individuals should be treated equally, regardless of race, sex, marital status, medical diagnosis, social standing, economic level, or religious belief.

D

An involuntarily committed client is verbally abusive to the staff, repeatedly threatening to sue. The client records the full names and phone numbers of the staff. Which nursing action is most appropriate to decrease the possibility of a lawsuit? A. Verbally redirect the client, and then refuse one-on-one interaction. B. Involve the hospital's security division as soon as possible. C. Notify the client that documenting personal staff information is against hospital policy. D. Continue professional attempts to establish a positive working relationship with the client. Rationale: The most appropriate nursing action is to continue professional attempts to establish a positive working relationship with the client. The involuntarily committed client should be respected and has the right to assert grievances if rights are infringed.

D

During hemodialysis, a client with chronic kidney disease develops headache, nausea, vomiting, and restlessness. After notifying the health care provider, which action by the nurse is most appropriate? a. Administer a bolus of dextrose solution. b. Draw blood for sodium and potassium. c. Order a blood urea nitrogen level stat. d. Prepare to administer phenytoin (Dilantin), Headache, nausea, vomiting, and restlessness may be signs of dialysis disequilibrium syndrome. Rapid decreases in fluid and in blood urea nitrogen (BUN) level can cause cerebral edema and increased intracranial pressure (ICP). Early recognition and treatment of this syndrome are essential for preventing a life-threatening situation. Treatment includes administration of anticonvulsants (Dilantin) or barbiturates. Dextrose is not used to treat disequilibrium syndrome, and sodium and potassium levels are not helpful because the symptoms are related to changes in urea levels and increased intracranial pressure. Obtaining the BUN would provide useful information; however, it is more important to treat the problem.

D

The RN has assigned a client with a newly placed arteriovenous (AV) fistula in the right arm to an LPN. Which information about the care of this client is most important for the RN to provide to the LPN? a."Avoid movement of the right extremity." b."Place gentle pressure over the fistula site after blood draws." c."Start any IV lines below the site of the fistula." d."Take blood pressure in the left arm." Repeated compression of a fistula site can result in loss of vascular access. Therefore, avoid taking blood pressures and performing venipunctures or IV placement in the arm with the vascular access. The other statements are not appropriate.

D

The client with end-stage cirrhosis presents with GI bleeding, combativeness, and confusion. The nurse anticipates an order to administer which medication? a. Omeprazole (Prilosec) b. Somatostatin (Octreotide) c. Propranolol (Inderal) d. Lactulose (Heptalac) Lactulose helps rid the body of ammonia. Excess ammonia leads to encephalopathy, which this client is manifesting. Omeprazole is a proton pump inhibitor used for reflux and ulcer disease. Somatostatin is given to treat bleeding from esophageal varices. Inderal is given to prevent bleeding from esophageal varices.

D

The nurse is attempting to position a client having an acute attack of pancreatitis in the most comfortable position possible. In which position does the nurse place this client? A Supine, with a pillow supporting the abdomen B Up in a chair between frequent periods of ambulation C High-Fowler's position, with pillows used as needed D Side-lying position, with knees drawn up to the chest The side-lying position with the knees drawn up has been found to relieve abdominal discomfort related to acute pancreatitis. No evidence suggests that supine position, sitting up in a chair, or high-Fowler's position have any effect on abdominal discomfort related to acute pancreatitis.

D

The nurse is caring for a client with acute pancreatitis. During the physical assessment, the nurse notes a grayish-blue discoloration of the client's flanks. Which is the nurse's priority action? a.Prepare the client for emergency surgery. b. Place the client in high Fowler's position. c. Insert a nasogastric (NG) tube to low intermittent suction. d.Ensure that the client has a patent large-bore IV site. Grayish-blue discoloration on the flanks (Turner's sign) indicates pancreatic enzyme leakage into the peritoneal cavity. This presents a risk of shock for the client, so IV access should be maintained with at least one large-bore patent IV catheter. The client may or may not need surgery; usually a fetal position helps with pain, and having an NG tube would not take priority over IV access.

D

The nurse is preparing to instruct a client with chronic pancreatitis who is to begin taking pancrelipase (Cotazym). Which instruction does the nurse include when teaching the client about this medication? A. Administer pancrelipase before taking an antacid. B. Chew tablets before swallowing. C. Take pancrelipase before meals. D. Wipe your lips after taking pancrelipase. Pancrelipase is a pancreatic enzyme used for enzyme replacement for clients with chronic pancreatitis. To avoid skin irritation and breakdown from residual enzymes, the lips should be wiped. Pancrelipase should be administered after antacids or histamine2 blockers are taken. It should not be chewed to minimize oral irritation and allow the drug to be released more slowly. It should be taken with meals and snacks and followed with a glass of water.

D

Which developmental characteristic should a nurse identify as typical of a client diagnosed with severe intellectual developmental disorder (IDD)? A. The client can perform some self-care activities independently. B. The client has more advanced speech development. C. Other than possible coordination problems, the client's psychomotor skills are not affected. D. The client communicates wants and needs by "acting out" behaviors. Rationale: The nurse should identify that a client diagnosed with severe IDD may communicate wants and needs by "acting out" behaviors. Severe IDD indicates an IQ between 20 and 34. Individuals diagnosed with severe IDD require complete supervision and have minimal verbal skills and poor psychomotor development.

D

Which statement by a client who has undergone kidney transplantation indicates a need for more teaching? a."I will need to continue to take insulin for my diabetes." b."I will have to take my cyclosporine for the rest of my life." c."I will take the antibiotics three times daily until the medication is finished." d."My new kidney is working fine. I do not need to take medications any longer." A crucial role of the nurse in long-term follow-up of the kidney transplantation client involves maintenance of prescribed drug therapy. Such clients will need to take immune suppressants for the rest of their lives to prevent rejection of the kidney.

D

Which statement should a nurse identify as correct regarding a client's right to refuse treatment? A. Clients can refuse pharmacological but not psychological treatment. B. Clients can refuse any treatment at any time. C. Clients can refuse only electroconvulsive therapy (ECT). D. Professionals can override treatment refusal by an actively suicidal or homicidal client. Rationale: The nurse should understand that health-care professionals could override treatment refusal when a client is actively suicidal or homicidal. A suicidal or homicidal client who refuses treatment may be in danger or a danger to others. This situation should be treated as an emergency, and treatment may be performed without informed consent.

D

For the patient with acute pancreatitis, monitor for significant changes in vital signs that may indicate the life-threatening complication of shock such as?

Hypotension and tachycardia may result from pancreatic hemorrhage, excessive fluid volume shifting, or the toxic effects of abdominal sepsis from enzyme damage. Observe the patient for changes in behavior and level of consciousness (LOC) that may be related to alcohol withdrawal, hypoxia, or impending sepsis with shock.

What causes Hepatocellular jaundice?

Develops because the liver cells cannot effectively excrete bilirubin. This decreased excretion results in excessive circulating bilirubin levels. Patients with jaundice often report pruritus (itching).

What is Separation Anxiety Disorder?

Excessive fear or anxiety concerning separation from those whom they are attached to. Exceeds what is expected for the norm of patient.

What is the Whipple Procedure and why is it done?

Extensive resection of part of head of pancreas, duodenum, stomach and gallbladder with anastomoses of the pancreatic duct., common bile duct, and jejunum, used to surgically treat Pancreatic Cancer.

What is fulminant hepatitis?

Failure of the liver cells to regenerate, with progression of the necrotic process, results in a severe acute and often fatal form of hepatitis. known as fulminant hepatitis.

Why does hypocalcemia occur in Acute Pancreatitis?

Fatty acids are released during this lipolytic process and combine with ionized calcium to form a soap-like product. The initial rapid lowering of serum calcium levels is not readily compensated for by the parathyroid gland. Because the body needs ionized calcium and cannot use bound calcium, hypocalcemia occurs.

When performing an abdominal assessment in a patient presenting with Acute Pancreatitis, inspect for what?

Generalized jaundice, gray-blue discoloration of the abdomen and periumbilical area & flanks, listen for absent or decreased bowel sounds, note abdominal tenderness, rigidity, and guarding because of peritonitis.

What is Diabetic ketoacidosis (DKA)?

It is characterized by uncontrolled hyperglycemia, (high blood sugar) metabolic acidosis, and increased production of ketones. This condition results from the combination of insulin deficiency and an increase in hormone release that leads to increased liver and kidney glucose production and decreased glucose use in peripheral tissues


Set pelajaran terkait

Unit 6: George Washington and John Adams

View Set

Chapter 5 Nervous System Exercise Phys.

View Set

Fill in the blank: A problem statement is a(n) _____.

View Set

Orthopedic Conditions FINAL (Ergonomics, THR/TKR, Chronic Pain)

View Set

AP World Multiple Choice on Unit 3

View Set

Ch. 5 TB: Marketing Research and Information Systems

View Set

MIGUEL AND GLENNA, A POP GROUP (4X4), SALLY'S FAMILY, SALLY'S BROTHER, MY FRIEND ANDY (*solo verbo essere*)

View Set

English final exam- authors/themes

View Set